小学英语第六册期中测试卷及答案
小学六年级上册第六次英语第六单元期中试卷[含答案解释]
小学六年级上册英语第六单元期中试卷[含答案解释]英语试题一、综合题(本题有50小题,每小题2分,共100分.每小题不选、错误,均不给分)1.My best friend, Lily, ______ (live) in a small town. She ______ (be) very friendly and ______ (always) help me with my homework. This week, she ______ (invite) me to her birthday party. I ______ (buy) her a gift, and I ______ (feel) excited about the party. I ______ (know) it ______ (be) a special day for her.2.Which of these is a pet?A. LionB. DogC. ElephantD. Tiger3.Today, we ______ (have) an art class. Our teacher ______ (show) us how to paint a landscape. First, we ______ (draw) the outline of a mountain, then we ______ (paint) the sky blue. After that, we ______ (add) some trees and a river. Everyone ______ (enjoy) the class because we ______ (learn) many new painting techniques.4.They _______ (not) play games on weekdays.5.What is the color of an apple?A. GreenB. BlueC. RedD. Pink6.What do we call the person who teaches us at school?A. DoctorB. TeacherC. ChefD. Driver7.I _______ (visit) my grandparents on weekends.8.Tom is going to the zoo with his class. His teacher tells the students that they will see many different animals. The first animal they see is a __________ (1). It has a long__________ (2) and big ears. Tom is amazed by how tall it is. Next, they visit the__________ (3), and the monkeys are jumping around and swinging from tree to tree. Tom laughs because the monkeys are very __________ (4). After that, they go to see the __________ (5). It has a beautiful mane and likes to sleep in the sun.9.Sarah ______ (study) for her math test. She ______ (use) her notebook and calculator. Right now, she ______ (work) on some difficult problems. Yesterday, she______ (practice) for an hour. Her brother ______ (help) her with the homework, and they ______ (finish) it together.10.I _______ my homework at this time yesterday.11.Which of these is a weather condition?A. SunnyB. BookC. ChairD. Table12.Every year, Jack’s family goes to the __________ (1) for a vacation. They usually stay in a small __________ (2) near the __________ (3). Jack enjoys walking along the __________ (4) and watching the __________ (5) in the sky. His little sister loves collecting __________ (6) from the beach. They also like to go swimming in the__________ (7) and have picnics on the __________ (8). Jack is already looking forward to next year’s __________ (9).13.Today, I _______ (have) a math test. I _______ (study) very hard for it. Before the test, my teacher _______ (review) the lesson with us. I _______ (feel) a little nervous, but I think I _______ (do) my best.14.My grandmother __________ (make) delicious cookies every Sunday. We__________ (visit) her on weekends, and I __________ (help) her in the kitchen. I__________ (like) baking with her because it __________ (be) fun.15.What do we use to read books?A. ScissorsB. GlassesC. CameraD. Spoon16.What color is the sky on a clear day?A. GreenB. BlueC. RedD. Yellow17.What is the opposite of hot?A. WarmB. ColdC. WetD. Dry18.When you are learning English, you need to practice speaking, listening, reading, and writing. But what is the best way to learn new words in English every day? Should you use a dictionary, read storybooks, write sentences, or just memorize random words without context?A. Use a dictionaryB. Read storybooksC. Write sentencesD. Memorize random words without context19.She _______ (play) with her friends in the park.20.My sister _______ a book.A. readsB. readC. is readD. reading21.I have a brother and a sister. My brother is younger than me, and my sister is older. We all like playing video games and watching movies together in the evening.22.Which of these is a season?A. WinterB. TuesdayC. JulyD. Bread23.I _______ (travel) to many places last summer. I _______ (visit) the beach, the mountains, and some historical sites. It _______ (be) a great adventure.24.Which of these animals is a mammal?A. FishB. BirdC. ElephantD. Snake25.Which of the following is a vehicle?A. AppleB. CarC. TreeD. Shirt26.Which of these is a number?A. DogB. FiveC. ChairD. Table27.My sister _______ like to play tennis.A. doesntB. dontC. isntD. arent28.Sally is baking a __________ for her mother’s birthday. She uses __________, sugar, eggs, and __________. After mixing the ingredients, she pours them into a__________ and bakes it in the oven. The cake looks __________ when it is ready.29.Which animal has a long trunk?A. ElephantB. DogC. LionD. Giraffe30.My mother is ______ teacher.A. AB. AnC. TheD. It31.Which one is a color?A. AppleB. BlueC. CatD. Chair32.I ______ (love) reading books. Every weekend, I ______ (visit) the library with my sister. Last Saturday, we ______ (find) a new book about dinosaurs. We ______ (read) it together and ______ (learn) many new things. It ______ (be) a very interesting book.33.Yesterday, I __________ (1) to the zoo with my friends. We __________ (2) many animals. First, we __________ (3) the lions. They __________ (4) very strong and fierce. Then, we __________ (5) the elephants. They __________ (6) big and gentle. After that, we __________ (7) to the monkey area and __________ (8) them playing with each other. It __________ (9) a fun day.34.What do we call the first meal of the day?A. DinnerB. LunchC. SnackD. Breakfast35.Today, we are learning about the weather in ______ class. We talked about the four seasons: ______, ______, ______, and ______. I like ______ because I can go skiing.My friend Sarah loves ______ because she enjoys the flowers blooming.36.Which of these is a time of day?A. MorningB. WinterC. AprilD. Chair37.Tomorrow, we __________ (have) a field trip to the zoo. We __________ (see) many animals, including lions, tigers, and giraffes. Our teacher __________ (tell) us that we __________ (learn) a lot about animal habitats and behavior. I __________ (look) forward to it because I __________ (love) animals!38.Jack and his family are planning a road trip this summer. They will drive to different __________ like nation al parks and museums. Jack’s father is going to drive the __________, and his mother will bring snacks and drinks for the trip. Jack and his sister, Lily, will bring their __________ to play with during the journey. Jack is very excited because he loves __________ and wants to see new places. He hopes they will stop at a __________ on the way to eat lunch.39.She _______ (not) have a sister.40.This afternoon, we __________ (1) on a field trip to the zoo. We __________ (2) many animals like lions, tigers, and elephants. My friend Emma __________ (3) to see the monkeys, but I __________ (4) to see the giraffes. It __________ (5) a fun trip.41.This afternoon, my friends and I __________ (1) to the park to play soccer. The weather __________ (2) very nice, and there __________ (3) a lot of people. We__________ (4) a great time. After the game, we __________ (5) some snacks and__________ (6) pictures.42.Which of these is used to cut?A. ForkB. SpoonC. KnifeD. Plate43.Which of these is a wild animal?A. TigerB. DogC. HorseD. Cow44.This weekend, we are going to have a family picnic at the __. We will bring a __, sandwiches, and __. I also packed a ball so we can play some games. I can’t wait to enjoy the __ weather and spend time with my family.45.What is the opposite of "clean"?A. DirtyB. WetC. SoftD. Heavy46.There _______ a cat on the table.A. isB. areC. beD. am47.Which of these is a body part?A. DogB. BookC. EarD. Table48.Which one is used for writing?A. PenB. KnifeC. PlateD. Fork49.Which animal has a long tail?A. DogB. MonkeyC. CatD. Elephant50.She _______ (is / are / was) going to the park now.(答案及解释)。
人教PEP版英语六年级上册6期中综合测试卷A含答案
2022-2023学年度上期六年级上册期中测试英语试题A 卷(人教PEP)Listening( 听力部分 )(40 分 )Ⅰ . 我会听录音 , 选择。
(8 分 )()1.AB ()2.A B()3.A B()4.A BⅡ . 我会听录音 , 选出听到的单词。
(12 分 )()1.A.cinema B.subway C.park ()e ually C.often ()3.A.dive B.drive C.live ()4.A.America B.tomorrow C.Australia ()5.A.colour B.road C.light ()6.A.space B.take C.birthdayⅢ . 我会听录音 , 填空。
(10 分 ) We must know the 1 rules.Remember,stop at the 2 3 ,wait at the 4 5 and go at the 6 7 .There are always 8 lights.In some countries drivers drive on the 9 side.In some other countries drivers drive on the 10 side.1.2. 3. 4. 5. _______6. 7. 8. 9. 10.Ⅳ . 我会听录音 , 判断正 ( √ ) 误 (×)。
(10 分 )()1.Chen Jie is going to the bookstore this Saturday.()2.She wants to buy a new CD and some picture books.()3.She is going by taxi.()4.She is going to read the new books on Saturday afternoon.()5.Mike is going to plant trees this weekend.Writing( 笔试部分 )(60 分)Ⅴ.我会判断。
小学六年级上册第六次英语第六单元期中试卷(答案和解释)
小学六年级上册英语第六单元期中试卷(答案和解释)英语试题一、综合题(本题有50小题,每小题2分,共100分.每小题不选、错误,均不给分)1.Tom is looking for his __________ in his room. He has lost it and needs to find it. After searching under the __________ and behind the __________, Tom finally finds his __________ in the drawer.2.Which of these is a type of tree?A. PineB. ChairC. SpoonD. Plate3.What is the opposite of "day"?A. MorningB. EveningC. NightD. Afternoon4.Which of these is a vegetable?A. SpinachB. BananaC. AppleD. Pear5.Which animal is known for its ability to fly?A. FishB. BirdC. ElephantD. Dog6.Which one is a body part?A. ArmB. SpoonC. TableD. Shoe7.How many months are there in a year?A. 10B. 12C. 11D. 98.Which one is a body part?A. LegB. TableC. PlateD. Spoon9.She ________ (not, like) running in the rain.10.What do we wear on our head?A. ShoesB. HatC. PantsD. Gloves11.Which one is a vehicle?A. CarB. PlateC. ChairD. Table12.We __________ (1) to the library every week. My sister __________ (2) a book about animals, and I __________ (3) a storybook. After we __________ (4) our books, we __________ (5) back home.13.What is the opposite of "fast"?A. SlowB. HeavyC. TallD. Short14.What do you use to write on a board?A. PenB. EraserC. ChalkD. Paper15.Jack loves reading books. His favorite books are about __________ and__________. He enjoys learning about how different animals live in the wild. Sometimes, Jack likes to imagine that he is a __________, exploring the jungle. He also enjoys reading about __________, especially how different plants grow. After reading, Jack likes to draw pictures of the animals and __________ he has read about in his books.16.I ________ (go) to school by bus.17.Which sentence is correct?A. The book is under the table.B. The book is at the table.C. The book is over the table.D. The book is on the table.18.I ______ (study) for my science test right now. I ______ (need) to learn about animals and plants. Yesterday, I ______ (watch) a documentary about the rainforest. I______ (learn) many new things, and I think I ______ (be) ready for the test tomorrow.19.We _______ lunch at 12:00 every day.20.They _______ (is / are / am) playing basketball now.21.What do we use to write?A. SpoonB. PenC. PlateD. Fork22.Which of these is a drink?A. WaterB. ShoeC. TableD. Carst weekend, I _______ (go) to the zoo with my parents. We _______ (see) many animals, including lions, elephants, and monkeys. It _______ (be) a wonderful day!24.Which of these is an animal that can swim?A. FishB. DogC. ElephantD. Bird25.I __________ (1) a new bicycle last month. It __________ (2) blue and white. Every morning, I __________ (3) to school by bicycle. It __________ (4) me only 10 minutes to get there. I __________ (5) riding my bicycle very much. I __________ (6) to go on bike trips with my friends.26.Which one is a vegetable?A. TomatoB. AppleC. CarrotD. Banana27.I ________ (not, have) any homework.28.They _______ to school by foot.29.Which of these is a day of the week?A. MondayB. JanuaryC. SummerD. Night30.What is the opposite of "big"?A. smallB. longC. tallD. fast31.My brother _______ (like) playing soccer with his friends. He practices every weekend, and he is getting better at it.32.I __________ (1) to the supermarket with my mom every week. She __________ (2) vegetables and fruits, and I __________ (3) to pick out snacks. We __________ (4) everything we need and __________ (5) home.33.Sarah loves reading books. She has many books in her __________ (1). Her favorite type of books are __________ (2) stories. Every evening, she reads a chapter before going to __________ (3). She also likes to read __________ (4) about animals and nature.34.They _______ in the garden now.A. workB. workedC. are workingD. is working35.Which animal is known as "mans best friend"?A. CatB. DogC. ElephantD. Tiger36.You are at the park. You see a slide, a seesaw, and a swing. You and your friends are playing together. What kind of place are you in?A. A museumB. A zooC. A playgroundD. A shopping mall37.Jack and his family are on vacation at the beach. They build a __________ (1) in the sand and collect some __________ (2) near the shore. Jack loves to __________ (3) in the water, while his sister enjoys making __________ (4) with her bucket.38.They ________ (be) friends for many years.39.Today, we ______ (have) a PE class. Our teacher ______ (teach) us how to play basketball. I ______ (enjoy) the class because I ______ (like) sports. My friends and I______ (play) a match. I ______ (be) the captain of the team. We ______ (win) the game in the end.40.Which of these is a vegetable?A. CarrotB. BananaC. PearD. Apple41.Which of these is used to eat spaghetti?A. KnifeB. SpoonC. ForkD. Plate42.Which of these is a sport?A. PaintingB. SoccerC. ReadingD. Sleeping43.Which of the following is a type of bird?A. SnakeB. RabbitC. SparrowD. Fish44.What is the opposite of "young"?A. FastB. OldC. TallD. Happy45.Which of these is used to clean your teeth?A. ToothbrushB. KnifeC. PlateD. Spoon46.My friend _______ very kind.47.Which of these animals is a pet?A. LionB. DogC. ElephantD. Tiger48.In English, we use modal verbs like "can," "should," and "must" to express ability, advice, or necessity. For example, "I can swim" expresses ability. Which of the following sentences uses "can" correctly?A. She can to play the piano.B. She can play the piano.C. She can plays the piano.D. She can playing the piano.49.David loves animals. His favorite animal is the __________. He often goes to the __________ with his parents to see different animals. Last week, they saw a __________ at the zoo. David also learned that __________ are very smart animals. His parents promised to take him to see more animals next __________.50.I ________ (not, like) vegetables.(答案及解释)。
小学六年级上册第六次英语第六单元期中试卷(含答案含解析)
小学六年级上册英语第六单元期中试卷(含答案含解析)英语试题一、综合题(本题有50小题,每小题2分,共100分.每小题不选、错误,均不给分)st summer, my family and I _______ (travel) to the beach. We _______ (stay) at a hotel near the sea. Every morning, we _______ (swim) in the ocean, and in the afternoon, we _______ (build) sandcastles on the beach. It _______ (be) an unforgettable vacation.2.Which sentence uses the correct subject pronoun for the word "Tom"?A. He is playing football.B. She is playing football.C. They is playing football.D. I is playing football.3.Which of these animals is a mammal?A. FishB. DogC. BirdD. Turtle4.What is the opposite of "empty"?A. FullB. HeavyC. LightD. Slow5.What do we use to cut fruit?A. SpoonB. ForkC. KnifeD. Plate6.Which of these is a shape?A. SquareB. AppleC. CarD. Chair7.Which of these is a number?A. AppleB. ThreeC. DogD. Spoon8.Which fruit is yellow?A. AppleB. BananaC. OrangeD. Grapest summer, my family __________ (1) go to the beach for vacation. We__________ (2) enjoy swimming in the sea. My brother __________ (3) like surfing. I __________ (4) not know how to surf, but I __________ (5) enjoy playing on the sand.10.Which of these is a question?A. I am hungry.B. You are my frienD.C. What is your name?D. This is my pencil.11.Which word is a color?A. BookB. RedC. CatD. Jump12.Jack is in the kitchen. He is making some __________ (1). He needs to add__________ (2) and some __________ (3). After mixing everything together, Jack pours the mixture into a __________ (4).13.We ________ (not, play) games at the moment.14.Emily and her friends are at the zoo. They are looking at the __________ (1), which is eating __________ (2). Emily notices that the __________ (3) is very tall and has a long __________ (4). Afterward, they visit the __________ (5) and watch the elephants play.15.Which word is a greeting?A. sorryB. thank youC. goodbyeD. hello16.Which animal says "woof"?A. CatB. DogC. CowD. Bird17.What color is an apple?A. GreenB. YellowC. RedD. Black18.She _______ her room every Saturday.19.Which one is a sea creature?A. DogB. CatC. FishD. Cow20.In the summer, my family and I __________ (1) to the beach. We __________ (2) swimming in the ocean and __________ (3) sandcastles on the shore. My sister__________ (4) to play with her toys, while I __________ (5) to collect seashells. After a long day, we __________ (6) back home and __________ (7) a big dinner.st summer, I ______ (visit) my cousin in the countryside. We ______ (go) swimming in the lake and ______ (pick) fresh fruit from the trees. I ______ (have) so much fun, and I ______ (wish) I could stay longer.22.Which of these is a body part?A. LegB. ChairC. SpoonD. Plate23.Which of the following is a fruit?A. BananaB. CarrotC. PotatoD. Lettuce24.I ______ (love) reading books. Last night, I ______ (read) a story about a pirate. It ______ (be) very exciting! After I ______ (finish) reading, I ______ (tell) my parents about the story.25.What is the opposite of "hot"?A. coldB. warmC. largeD. dry26.I ____ to the store yesterday.A. goB. goingC. wentD. goes27.What do we use to brush our teeth?A. BrushB. SpoonC. ForkD. Knife28.Which of these is a mode of transport?A. BusB. SpoonC. TableD. Chair29.I have a _______ dog.A. smallB. catC. happyD. teacher30.We ______ (celebrate) my mother’s birthday last Sunday. She ______ (get) many presents from us. We ______ (have) a big cake, and we ______ (sing) a birthday song. After the party, we ______ (go) to a restaurant for dinner. It ______ (be) a very happy day.31.Which of the following is a type of transportation?A. BallB. CarC. BookD. Chair32.Which one is a time of day?A. WinterB. MorningC. GreenD. Hot33.Jack is playing __________ with his friends at the park. They are having fun kicking the ball back and forth. Jack tries to __________ the ball into the goal, but his friend stops it. After playing, they sit on the __________ and eat some __________. Jack loves playing __________ because it is exciting and fun.34.What is the opposite of “big”?A. smallB. tallC. heavyD. light35.Which one is a body part?A. HandB. SpoonC. BookD. Chair36.Which of these is a musical instrument?A. GuitarB. CarC. PlateD. Chair37.Ben is going to the store with his dad. They need to buy some __________ (milk), __________ (bread), and __________ (eggs). Ben also picks out a __________ (toy) and a bag of __________ (chips) for his snack. After paying, they leave the store and head home in their __________ (car).38.Which of these is a wild animal?A. CatB. DogC. LionD. Cow39.What is the opposite of "clean"?A. DirtyB. WetC. TallD. Small40.This morning, I __________ (wake) up late because I __________ (stay) up too late last night. I __________ (rush) to get ready, but I __________ (miss) the bus. I__________ (run) to school and __________ (arrive) just in time for the first lesson. During the lesson, I __________ (pay) attention to the teacher, and I __________ (finish) my homework quickly. After school, I __________ (go) to the library to study.41.Which of the following animals live in water?A. ElephantB. DogC. FishD. Lion42.This morning, I __________ (get) up early because I __________ (want) to finish my homework. After I __________ (eat) breakfast, I __________ (study) for my math test. I __________ (find) the test a little difficult, but I __________ (do) my best. In the afternoon, I __________ (take) a walk to relax after studying.43.We _______ doing our homework now.44.Jack is learning how to play the guitar. He practices every __________ (1). His teacher shows him how to play a __________ (2) and teaches him the __________ (3). Jack is very __________ (4) because he loves music.45.What is the opposite of "young"?A. OldB. TallC. HeavyD. Light46.What is the color of the sky on a clear day?A. RedB. YellowC. BlueD. Green47.Which of these is a shape?A. SquareB. CarC. ChairD. Table48.This is my friend Jack. He __________ (1) very tall. Jack __________ (2) love playing basketball, and he __________ (3) practice every afternoon. He __________ (4) go to the gym after school. His favorite color __________ (5) blue, and his favorite food __________ (6) pizza. Jack __________ (7) always helps me with my homework. 49.Fill in the blank with the correct form of the adjective: Sally is very good at painting, but her brother is _______ (good) at it than she is. She wants to get better, so she practices _______ (hard) every day. Sally thinks that with more effort, she can become _______ (good) than her brother.50.There is a __ in the kitchen. I use it to cook food. It has four burners, and it is very useful for making meals. My mom usually cooks dinner for the family, but sometimes I help her. I like learning how to cook simple dishes.(答案及解释)。
小学六年级上册第六次英语第六单元期中试卷(答案和解释)
小学六年级上册英语第六单元期中试卷(答案和解释)英语试题一、综合题(本题有50小题,每小题2分,共100分.每小题不选、错误,均不给分)st Saturday, my family __________ (go) to the park. We __________ (bring) a picnic with us. The weather __________ (be) sunny and warm, so we __________ (sit) on the grass and __________ (enjoy) the day. My brother __________ (play) football with his friends while I __________ (read) a book. We __________ (have) a great time together. After the picnic, we __________ (take) some photos and __________ (leave) the park at 5 p.m.2.Which of these is a holiday?A. ChristmasB. MondayC. SummerD. Book3.I __________ (love) reading books. Every week, I __________ (borrow) books from the library. Last week, I __________ (read) a mystery book and __________ (enjoy) it very much.4.Which one is a holiday?A. ChristmasB. MondayC. SummerD. Winter5.Lily is very creative. She loves to draw and paint. Today, she is working on a picture of a beautiful __________ (1). She carefully draws the outline of the tree and then adds __________ (2) to make the tree look realistic. Lily uses her favorite color, green, to color the leaves, and brown for the __________ (3). After finishing the tree, Lily adds some __________ (4) in the background to make her picture look like a forest. Finally, she draws a bright __________ (5) in the sky.6.Tomorrow, we __________ (have) a field trip to the zoo. We __________ (see) many animals, including lions, tigers, and giraffes. Our teacher __________ (tell) us that we __________ (learn) a lot about animal habitats and behavior. I __________ (look) forward to it because I __________ (love) animals!7.What do we use to write on paper?A. EraserB. PenC. ScissorsD. Knife8.Which one is correct?A. My father is tall.B. My father are tall.C. My father tall.D. My father being tall.9.Which of these is a type of food?A. ShoeB. AppleC. ShirtD. Car10.What do you use to write on paper?A. EraserB. PencilC. RulerD. Paper11.My friend Lily ______ (be) a very good singer. She ______ (sing) in the school choir every week. Last month, she ______ (perform) at the school concert. She ______ (sing) a beautiful song and everyone ______ (clap) after her performance. Lily ______ (be) very happy because she ______ (do) a great job.12.What do we use to play music?A. GuitarB. SpoonC. PlateD. Knife13.My teacher __________ (1) English very well. She __________ (2) us new words every day. Yesterday, she __________ (3) a story, and we __________ (4) to answer questions about it. I __________ (5) the story because it __________ (6) very interesting.14.Which of these is the opposite of "fast"?A. SlowB. TallC. HeavyD. Big15.Peter ______ (like) playing basketball. He ______ (play) with his friends every afternoon. Yesterday, they ______ (play) for two hours and ______ (score) many points. After the game, they ______ (rest) for a while and ______ (talk) about the game.16.What is the plural of "book"?A. BookesB. BooksC. BookD. Bookes17.This morning, we ______ (have) a science lesson at school. Our teacher ______ (teach) us about the solar system. We ______ (learn) that the Earth ______ (orbit) the Sun and that there ______ (be) eight planets in our solar system. It ______ (be) a very interesting class.18.What is the opposite of "big"?A. SmallB. HeavyC. LongD. Fast19.Which one is correct?A. I am a teacher.B. I are a teacher.C. I be a teacher.D. I teacher am.20.My dad is a doctor. He works in a hospital. Every day, he helps sick people feel better. Sometimes, he works very long hours, but he enjoys his job. He told me that being a doctor is very rewarding because he can make a difference in people’s lives. When I grow up, I want to help others too, just like my dad. I think I will become a nurse.21.Which of these is a mammal?A. SnakeB. ElephantC. FishD. Bird22.Lily is reading a __________ (1) about animals. She loves __________ (2) because they are so __________ (3) and __________ (4). Her favorite animal is the __________ (5), and she hopes to see one in the zoo.23.The birds _______ (fly) south for the winter every year. They travel long distances to find warmer places.24.Which one is a season?A. MondayB. WinterC. WaterD. School25.Which animal can fly?A. DogB. CatC. BirdD. Elephant26.Which of these is a day of the week?A. JanuaryB. SundayC. SummerD. Christmas27.Which animal is known for saying "moo"?A. CowB. SheepC. DogD. Cat28.Which of these is a classroom item?A. TowelB. BookC. KnifeD. Table29.Yesterday, we __________ (go) to the beach. The sun __________ (shine) brightly, and the sky __________ (be) clear. My brother and I __________ (swim) in the ocean, while our parents __________ (lie) on the beach and __________ (read) books. After swimming, we __________ (build) a big sandcastle. It __________ (be) a perfect day at the beach.30.Tom loves playing video games. He usually plays with his __________ on weekends. They like to play games like __________, __________, and __________. Tom’s favorite game is __________, which is a racing game. He always tries to__________ and win the race, but sometimes he gets a little __________ when he loses. Tom also enjoys watching his favorite __________ on YouTube, where he learns newtricks to improve his gameplay. Tom hopes to become a better player and maybe even__________ in a tournament one day.st weekend, I __________ (1) to a concert with my family. It __________ (2) a great experience! The music __________ (3) so beautiful, and the singer __________ (4) very talented. We __________ (5) for two hours and __________ (6) many songs. After the concert, we __________ (7) to a restaurant to have dinner.32.We __________ (1) to the mountains last summer. We __________ (2) many beautiful flowers and __________ (3) a picnic by the river. I __________ (4) to visit the mountains again next year.33.Which one is a time of day?A. JanuaryB. WinterC. MorningD. December34.My brother and I _______ (decide) to clean the garage last weekend. First, we_______ (move) all the boxes out, and then we _______ (sweep) the floor. After that, we _______ (organize) the tools and _______ (put) everything in its place. We _______ (find) an old bike that we _______ (forget) about. It _______ (be) a lot of work, but we_______ (feel) proud when we _______ (finish) cleaning. My parents _______ (be) very happy with the result.35.I ______ (not/like) vegetables, but I ______ (love) fruits. Yesterday, I ______ (eat) an apple and a banana. My sister ______ (eat) some carrots and tomatoes. She ______ (like) them very much.36.Which of these is a pet?A. ChickenB. DogC. CowD. Sheep37.I __________ (1) my homework every afternoon. After I __________ (2) it, I usually __________ (3) some time to read a book. Yesterday, I __________ (4) a story about a robot. It __________ (5) a great story, and I __________ (6) to finish reading it today.38.Anna is reading a book in the library. She loves to read about __________. She enjoys learning about different animals and their __________. After reading for a while, Anna takes a break and goes to the __________ to borrow a new book about __________.39.What do we use to eat pasta?A. KnifeB. ForkC. SpoonD. Chopsticks40.This morning, I __________ (wake) up early because I __________ (have) an important exam. I __________ (eat) breakfast quickly and __________ (leave) the house. On my way to school, I __________ (see) my friend Anna. We __________ (walk) to school together. We __________ (arrive) just on time for the exam.41.David is cleaning his room. First, he picks up his __________ (1) and puts them in the __________ (2). Then, he uses a __________ (3) to clean the floor. He also puts his __________ (4) back in their place. After finishing, David feels __________ (5).42.What time is it when the clock shows 3:00?A. 3 oclockB. 12 oclockC. 6 oclockD. 9 oclock43.Which of these animals lives in the water?A. DogB. BirdC. FishD. Elephant44.Which one is a number?A. FiveB. SpoonC. PlateD. Chair45.Anna is at the zoo. She sees a __________ (1) walking slowly, a __________ (2) jumping around, and a __________ (3) swimming in the water. Anna takes a__________ (4) of the animals.46.How many legs does a spider have?A. sixB. sevenC. eightD. ten47.My sister and I ______ (play) badminton every weekend. Last Sunday, we ______ (play) for two hours. After that, we ______ (eat) lunch together and ______ (talk) about our plans for the next holiday.48.Which of these is an animal that lives in the ocean?A. ElephantB. FishC. DogD. Lion49.Jack and his family are going on a trip. They pack their __________ into the car and drive to the mountains. On the way, they see some __________ on the road. Jack takes out his __________ and starts taking pictures.50.Every year, my family and I ______ (go) on a trip. Last summer, we ______ (visit) the beach. We ______ (stay) there for a week. The weather ______ (be) sunny and warm.I ______ (swim) in the sea and ______ (build) a sandcastle. We also ______ (eat) ice cream and ______ (take) many photos. It ______ (be) a great holiday.(答案及解释)。
【精品】人教版小学英语六年级上册期中试卷含参考答案
一、选择题(每题2分,共20分)1. 选择正确的单词填空。
( ) A. book B. pencil C. ruler2. 选择正确的动词填空。
( ) A. am B. is C. are3. 选择正确的介词填空。
( ) A. in B. on C. under4. 选择正确的形容词填空。
( ) A. happy B. sad C. angry5. 选择正确的副词填空。
( ) A. always B. sometimes C. never二、填空题(每题2分,共20分)1. My name is ________. I am ________ years old.2. I have two ________. They are ________ and ________.3. There are ________ books on the desk.4. I like ________ football. Do you like ________?5. She is a ________. She teaches ________.三、改错题(每题2分,共10分)1. I am a student. I like reading books.2. He have a pen. It is blue.3. She is eating. She is not drinking.4. We go to school bus. We like it.5. They are playing basketball. They are not running.一、选择题1. A2. B3. C4. A5. C二、填空题1. Tom ten2. brothers Lucy Lily3. three4. playing playing5. teacher English三、改错题1. 正确2. has3. is eating is drinking4. 正确5. is playing basketball is running四、完形填空(每题2分,共20分)阅读下面的短文,根据短文内容,选择正确的选项填空。
外研版英语小学六年级上学期期中试题与参考答案(2024-2025学年)
2024-2025学年外研版英语小学六年级上学期期中复习试题与参考答案一、听力部分(本大题有12小题,每小题2分,共24分)1、What is the name of the school where Tom goes to study?A. Foreign Language SchoolB. Happy English SchoolC. New Century Primary SchoolD. English Village Primary SchoolAnswer: CExplanation: The question asks for the name of the school where Tom studies. The correct answer can be found in the context provided, which states that Tom goes to “New Century Primary School.”2、How does the teacher usually start the morning class?A. By asking students to read the textbookB. By singing a song with the classC. By reviewing the homeworkD. By giving a short speechAnswer: BExplanation: The question inquires about the teacher’s usual morning classactivity. The correct answer is given as the teacher starts with “singing a song with the class,” which is a common practice to engage students and createa positive learning environment.3、What are the two animals that the teacher mentioned in the story?A. A rabbit and a turtleB. A wolf and a sheepC. A lion and a deerD. A snake and a frogAnswer: AExplanation: The teacher in the story mentioned that the rabbit and the turtle are the two animals that are good friends. Therefore, the correct answer is A.4、How does the little girl feel about her birthday party?A. She is happy but a little sad.B. She is excited but a bit nervous.C. She is sad because she couldn’t go.D. She is excited and looking forward to it.Answer: DExplanation: In the story, the little girl is described as feeling very excited and looking forward to her birthday party. There is no mention of sadness or nervousness, so the correct answer is D.5.Listen to the following conversation and answer the question.A: Hello, how are you doing today, Emily?B: I’m doing fine, thanks. How about you, Mr.Smith?A: I’m good, thanks. I was just wondering if you’ve finished your science project.B: Yes, I finished it yesterday. I’m quite proud of it.A: That’s great to hear. What was your project about?B: It was about the solar system. I learned so much about the planets and their characteristics.A: That sounds fascinating. Did you enjoy doing the project?B: Absolutely, it was a lot of fun. I spent a lot of time researching and experimenting.Question: What was Emily’s science project about?A) The weatherB) The solar systemC) The human bodyD) The animal kingdomAnswer: B) The solar systemExplanation: The answer can be found in the conversation when Emily says, “It was about the solar system. I learned so muc h about the planets and their characteristics.”6.Listen to the following dialogue and complete the sentence with the correct information.A: Hi, Sarah! Have you been to the new amusement park in our town yet?B: Yes, I went there last weekend. It was so much fun!A: Really? What kind of rides did you enjoy the most?B: I loved the roller coaster and the Ferris wheel. They were both thrilling.A: Oh, I can’t wait to go. I heard they have a haunted house too.B: Yes, they do. But be prepared for some spooky surprises!A: Spooky surprises, huh? I’ll be sure to take a friend with me.Question: What did Sarah say she loved the most about the amusement park?A)The haunted houseB)The roller coasterC)The Ferris wheelD)The food courtAnswer: B) The roller coasterExplanation: The answer is found in the dialogue when Sarah says, “I loved the roller coaster and the Ferris wheel. They were both thrilling.”7、Listen to the following dialogue and answer the question.W: Hi, John. How was your science project?M: It was great! I did a report on the solar system. What about you, Lily?W: I did a project on animals. I really enjoyed it.Question: What project did John do?A. A report on the solar systemB. A project on animalsC. A project on spaceD. A project on plantsAnswer: AExplanation: The question asks what project John did. In the dialogue, John says, “I did a report on the solar system,” so the correct answer is A.8、Listen to the following passage and fill in the blanks with the correct words.If you want to improve your English, you need to practice speaking and listening. The best way to do this is to talk to native speakers. You can find them in your city or even online. Another great way to practice is to watch English movies or TV shows. This will help you get used to the pronunciation and intonation. And don’t forget to read books in English. It will expand your vocabulary and improve your reading skills.Blank 1:_______to improve your English, you need to practice speaking and listening.Blank 2:_______way to do this is to talk to native speakers.Blank 3:_______great way to practice is to watch English movies or TV shows. Blank 4:_______you read books in English, it will expand your vocabulary and improve your reading skills.Answer:Blank 1: IfBlank 2: TheBlank 3: AnotherBlank 4: IfExplanation: The passage provides tips on how to improve English skills. The blanks need to be filled with the correct words to complete the sentences. The correct answers are “If,” “The,” “Another,” and “If,” respectively.9.You hear a conversation between two students, Tom and Lily, about their weekend plans.Tom: Hey Lily, what are you doing this weekend?Lily: Well, I’m going to visit my grandparents in the countryside. How about you?Tom: That sounds nice. I’m planning t o go to the museum with my parents.Question: What is Tom going to do this weekend?A) Visit grandparentsB) Go to the museumC) Spend time with friendsD) Go to the countrysideAnswer: B) Go to the museumExplanation: In the conversation, Tom says he is planning to go to the museum with his parents, which indicates that his weekend plan is to visit the museum.10.You hear a short dialogue between a teacher and a student, Alice, discussing a school project.Teacher: Alice, how is your science project coming along?Alice: It’s going well, Mr.Smith. I’ve already gathered some data and I’m almost finished with the experiment.Teacher: That’s great, Alice. Remember to include all the necessary information in your report.Question: What is Alice currently working on?A) A history projectB) A science experimentC) A mathematics problemD) A literature analysisAnswer: B) A science experimentExplanation: Alice mentions that she is working on her science project and has almost finished the experiment, indicating that she is involved in a science-related project.11.Listen to the conversation between two students and answer the question.A. What are the two students mainly discussing?1.Their favorite subjects.2.The school’s sports day.3.The upcoming school trip.Answer: 3. The upcoming school trip.Explanation: In the conversation, the two students talk about the preparations for the upcoming school trip, which is the main topic of their discussion.12.Listen to the dialogue and choose the correct option to complete the sentence.B. Why does the teacher ask the students to write a story?1.Because they want to practice their writing skills.2.Because they need to finish the assignment by the end of the week.3.Because they are going to participate in a writing competition. Answer: 1. Because they want to practice their writing skills.Explanation: The teacher asks the students to write a story as a way to help them practice their writing skills, not because of an assignment or competition.二、选择题(本大题有12小题,每小题2分,共24分)1、Choose the correct word to complete the sentence.The cat is sitting under the tree, feeling quite _______.A. happyB. hungryC. tiredAnswer: AExplanation: The sentence describes the cat as sitting under the tree, which suggests it is feeling relaxed and content, indicating a sense of happiness.2、Complete the sentence with the correct form of the verb in parentheses.The dog (bark) loudly when it saw the stranger.A. barksB. barkedC. is barkingExplanation: The sentence describes an action that has already occurred in the past. Therefore, the correct form of the verb is the simple past tense, “barked.”3、Choose the correct word to complete the sentence.The cat was (A) sleeping (B) eating (C) playing in the garden.答案:A解析:根据句意“那只猫正在睡觉”,应选择A项sleeping。
小学六年级上册第6次英语第六单元期中试卷(含答案含解析)
小学六年级上册英语第六单元期中试卷(含答案含解析)英语试题一、综合题(本题有50小题,每小题2分,共100分.每小题不选、错误,均不给分)1.Which of these animals is a mammal?A. FishB. BirdC. ElephantD. Reptile2.Yesterday, we __________ (go) to the museum to see the new dinosaur exhibition. It __________ (be) amazing! We __________ (learn) a lot about dinosaurs and their fossils. After the tour, we __________ (take) some pictures in front of the giant dinosaur model.I __________ (feel) very excited because it __________ (be) my first time visiting the museum.3.What do you use to eat soup?A. SpoonB. KnifeC. ForkD. Plate4.Which of these is a day of the week?A. MondayB. SummerC. DecemberD. January5.They _______ (go) to the park on Sundays.6.Which of these is a wild animal?A. DogB. CowC. TigerD. Elephant7.What is the opposite of "day"?A. NightB. MorningC. AfternoonD. Evening8.What is the weather like today? It’s ________.A. rainyB. rainC. to rainD. raining9.He ________ (do) his homework at this moment.10.Which one is a body part?A. FingerB. PlateC. SpoonD. Chair11.What do we use to cut paper?A. ScissorsB. KnifeC. SpoonD. Fork12.The bird can _______.A. flyB. flyingC. fliesD. flew13.We have a big __ in our neighborhood. It has many shops where you can buy __, clothes, and toys. There is also a __ where you can watch movies. Sometimes, we go there to eat at a __. I love visiting the mall with my family because we always have a good time.14.She _______ a letter to her friend last week.15.I ________ (go) to the beach last summer. We ________ (swim) in the sea and________ (build) sandcastles. It ________ (be) very hot, but we ________ (have) a lot of fun.16.Which of these is an animal that hops?A. DogB. ElephantC. FrogD. Cat17.I ______ (take) a shower every morning before school. After I ______ (finish), I______ (get) dressed and ______ (eat) breakfast. This morning, I ______ (eat) cereal with milk. It ______ (be) very delicious, and I ______ (feel) ready for the day.18.Which of these is a month of the year?A. JanuaryB. SummerC. AprilD. Day19.Which of these is a vegetable?A. BananaB. CucumberC. PearD. Strawberry20.In my free time, I like to read books. My favorite type of book is __. I recently reada book about a group of friends who went on an adventure to find a __. They faced many challenges, but in the end, they found the __ hidden in a cave.21.My parents _______ in the city.22.I _______ (have) a pet cat.23.They _______ not want to play with me.24.We _______ a movie last night.25.Which of the following is a fruit?A. CarrotB. BananaC. PotatoD. Lettuce26.I love animals, especially __. They are very __ and friendly. I also like to visit the __ to see other animals like __, __, and __. Sometimes, I watch documentaries about animals and learn more about them.27.Which of these animals has a long tail?A. DogB. ElephantC. CatD. Rabbit28.Which animal is the largest?A. DogB. ElephantC. WhaleD. Tiger29.Which of these is a shape?A. SquareB. DogC. ChairD. Table30.Which of these is a mode of transportation?A. CarB. ChairC. PlateD. Tshirt31.Sarah ______ (be) very excited because she ______ (get) a new bike for her birthday. She ______ (ride) it around the neighborhood, and all her friends ______ (join) her. They ______ (have) a great time riding together.32.I ______ (play) the piano every day after school. My mother ______ (teach) me how to play, and I ______ (practice) for about 30 minutes each time. I ______ (enjoy) playing music because it ______ (make) me feel happy. Sometimes, my friends ______ (come) over to listen to me play. They ______ (like) the music, and we always ______ (have) a good time together.33.I am ________ than you.A. tallerB. tallC. tallestD. the tallest34.He _______ to the park yesterday afternoon.35.I __________ (have) a pet dog named Max. He __________ (be) very friendly and __________ (love) playing with me. Every morning, I __________ (take) him for a walk, and in the evening, we __________ (play) fetch in the yard.36.My brother Jack __________ (1) always get up at 7:00 a.m. every day. After he__________ (2) get up, he __________ (3) take a shower. Then he __________ (4) eat breakfast at 7:30 a.m. He __________ (5) go to school at 8:00 a.m. and __________ (6)come back at 4:00 p.m. After school, he __________ (7) do his homework and__________ (8) play with his friends.37.This morning, I __________ (forget) to bring my homework to school. When I__________ (realize) it, I __________ (ask) my friend to help me. He __________ (lend) me his homework, so I __________ (not/get) into trouble.38.My favorite season __________ (1) summer. In summer, the weather __________ (2) usually hot, and the days __________ (3) long. I __________ (4) like summer because I __________ (5) go swimming in the pool. My family __________ (6) often go on a trip during the summer holidays. We __________ (7) visit the beach or the mountains. I __________ (8) always have a great time in summer.39.Which of these is a cold drink?A. TeaB. WaterC. SoupD. Coffee40.Which of these is a shape?A. ChairB. TriangleC. TableD. Pen41.Which animal can swim?A. DogB. ElephantC. FishD. Giraffe42.I ______ (like) to eat fruits, especially apples and bananas. Yesterday, I ______ (eat) an apple and a banana for breakfast. This morning, I ______ (eat) some eggs with toast. I always ______ (drink) orange juice with my meals.43.Which of these is a fruit?A. TomatoB. OnionC. CarrotD. Potato44.Which of these is a pet?A. ChickenB. DogC. CowD. Sheep45.I _______ (visit) my grandmother last weekend. She _______ (live) in a small village. We _______ (walk) around the village, and my grandmother _______ (show) me her beautiful garden. After that, we _______ (sit) down for lunch, and she _______ (make) my favorite soup.46.Which of these is a vehicle?A. CarB. ChairC. SpoonD. Plate47.Yesterday, I __________ (go) to the library with my friend Amy. We __________ (arrive) there at 10 a.m. and __________ (start) looking for books. Amy __________ (find) a book about animals, and I __________ (choose) a book about space. We__________ (read) for two hours, and then we __________ (decide) to go to the park. The weather __________ (be) perfect, so we __________ (play) soccer for a while before going home.48.Which of these animals can fly?A. ElephantB. BirdC. DogD. Cow49.If it is raining outside, what should you bring with you?A. SunglassesB. An umbrellaC. A towelD. A swimsuit50.What do you use to eat soup?A. ForkB. KnifeC. SpoonD. Plate(答案及解释)。
冀教版英语小学六年级上学期期中试题与参考答案(2024年)
2024年冀教版英语小学六年级上学期期中复习试题与参考答案一、听力部分(本大题有12小题,每小题2分,共24分)1、What are the children doing in this picture?A. They are playing soccer.B. They are playing basketball.C. They are playing chess.Answer: AExplanation: The picture shows a group of children wearing soccer jerseys, witha soccer ball in the foreground, indicating that they are playing soccer.2、Listen to the dialogue and answer the question.M: How was your weekend, Li Ming?W: It was great, thanks! I went hiking with my family.How did Li Ming spend his weekend?A. He went hiking.B. He went shopping.C. He stayed at home.Answer: AExplanation: In the dialogue, Li Ming says, “It was great, thanks! I went hikingwith my family,” which clearly indicates that he spent his weekend hiking.3、What is the weather like in the city where the speaker is visiting?A. sunnyB. cloudyC. rainyD. windyAnswer: AExplanation: The speaker mentions that the weather is very sunny and warm, which indicates that the answer is A. sunny.4、Why is the speaker late for school today?A. They had a traffic jam on the way.B. Their alarm clock didn’t work.C. They had a lot of homework to finish.D. They were stuck in the elevator.Answer: AExplanation: The speaker explains that they were stuck in a traffic jam on the way to school, which is why they are late. Therefore, the correct answer is A. They had a traffic jam on the way.5、Listen to the conversation between two students in the library and answer the question.A. What are the two students mainly talking about?B. Why does the girl want to borrow the book?Answer: A. What are the two students mainly talking about?Answer: B. Why does the girl want to borrow the book?解析:这道题目考查听力理解能力。
小学上册第六次英语第六单元期中试卷(有答案)
小学上册英语第六单元期中试卷(有答案)英语试题一、综合题(本题有100小题,每小题1分,共100分.每小题不选、错误,均不给分)1.The _____ (大象) has a long trunk.2.I have ________ brothers and sisters.3.An oak tree is a type of ______ (树).4.Dolphins are very ______ (聪明) animals that live in the ocean.5.What is the capital of Indonesia?A. JakartaB. BaliC. SurabayaD. Bandung答案:A6.I see _____ (星星) at night.7.The chemical formula for methane is __________.8.The first man on the moon was _____.9.The fox is very ______ (机灵) and clever.10.I saw a _______ (狐狸) in my backyard last night.11.My sister is my best _______ because we share everything together.12.I love to watch ______ while I eat dinner.13.The __________ (历史的延续性) ensures legacy.14.What do you call the sound made by a dog?A. MeowB. MooC. BarkD. Roar答案: C15.The pig snorts when it is _________. (开心)16. A ______ (马) can be very strong and fast.17. A solution that does not conduct electricity is called a ______ solution.18.The __________ is a large lake in East Africa. (维多利亚湖)19.We watch ___ (cartoons/movies) on Saturday.20.My cousin is a great __________ (公关专家).21. A ferret loves to ______ (玩耍) and explore.22.The cat chases a _____ butterfly.23.We enjoy playing in the ___. (snow)24. A cat has excellent ______ (视力) at night.25.I always check the ______ (天气预报) before leaving home.26.My friend is very __________ (忠诚的) to our friendship.27.The _______ (Punic Wars) were fought between Rome and Carthage.28.The ________ (家庭活动) strengthen bonds.29.The ability of a substance to dissolve in water is called ________.30.What is the main ingredient in a Caesar salad?A. LettuceB. SpinachC. KaleD. Bok Choy答案:A31.My favorite animal is a ______ (猫) that loves to chase toys.32.The __________ is known for its deserts and unique wildlife. (澳大利亚)33.Chemical reactions are often represented by ______ equations.34.What do we call the process of making cloth from threads?A. WeavingB. KnittingC. StitchingD. Crocheting答案: A. Weaving35.What do we call the main character in a story?A. ProtagonistB. AntagonistC. NarratorD. Author答案: A36.The ______ of a tree is often wider than its trunk. (树的冠层通常比树干更宽。
人教版英语小学六年级上学期期中试题及答案指导(2024年)
2024年人教版英语小学六年级上学期期中复习试题及答案指导一、听力部分(本大题有12小题,每小题2分,共24分)1、听力原文:W: Good morning, Class 6! Can you tell me your name, please?M: Hello, Miss. I’m Li Ming.Question: What is the student’s name?A) Li MingB) Li MeiC) Li HongD) Li JianAnswer: A) Li MingExplanation: The student introduces himself as “Li Ming,” which is clearly stated in the dialogue. Therefore, the correct answer is A) Li Ming.2、听力原文:W: Excuse me, are you going to the library after school?M: Yes, I am. I need to borrow some books for my science project.Question: Why is the boy going to the library?A) To borrow books for a science projectB) To meet his friendsC) To study for a math testD) To read a novelAnswer: A) To borrow books for a science projectExplanation: The boy explicitly states that he needs to borrow books for his science project. Therefore, the correct answer is A) To borrow books fora science project.3、Listen to the following dialogue and choose the best answer for each question.A. What is the weather like today?B. How do you go to school?C. What are you doing after school?D. How much is the book?Answer: CExplanation: The question asks about what the speaker is doing after school. The correct answer can be found in the dialogue where the speaker mentions, “After school, I am going to the library.”4、Listen to the passage about a famous landmark and answer the following question.What is the name of the famous landmark in Paris?A. The Tower of LondonB. The Eiffel TowerC. The ColosseumD. The Great WallAnswer: BExplanation: The passage clearly mentions that the famous landmark in Paris is the Eiffel Tower, making option B the correct answer. The other options are landmarks in other cities.5、Listen to the conversation and answer the question.W: Hi, Tom. How was your weekend?M: Oh, it was great! I went hiking with my family. We saw a lot of beautiful scenery and even spotted some wildlife.Question: What did Tom do over the weekend?A) He went to the beach.B) He went hiking.C) He stayed at home.Answer: BExplanation: The question asks what Tom did over the weekend. In the conversation, Tom mentions that he went hiking with his family, so the correct answer is B) He went hiking.6、Listen to the dialogue and fill in the blanks with the correct words. W: Hi, John. I heard you joined the school’s science club. That’s great! M: Yes, it is. I’m really excited about it. We’re going to have a special presentation next week.W: Oh, that sounds interesting. What will you be presenting?M: Well, we’re going to talk about the solar system. I’m responsible for explaining about the planets.W: That’s fascinating! I always wanted to learn more about space.M: You should definitely join us next week. It’s going to be a lot of fun.Question: What are they planning to present about?A) The history of space exploration.B) The solar system.C) The mysteries of the universe.Answer: BExplanation: The dialogue clearly states that they are going to talk about the solar system. Therefore, the correct answer is B) The solar system.7.Listen to the following conversation and choose the best answer.A. What is the weather like today?B. Where are they going for vacation?C. How are they planning to spend their weekend?Answer: BExplanation: In the conversation, the speakers are discussing their vacation plans, which indicates that the topic of the conversation is about where they are going for vacation.8.Listen to the following dialogue and answer the question.Question: What is the main reason the woman can’t attend the party?A. She has a cold.B. She has to work late.C. She is going on a trip.Answer: BExplanation: The woman explicitly mentions that she has to work late, which is the reason she cannot attend the party. This makes option B the correct answer.9.Listen to the following dialogue and choose the best answer to complete the sentence.A: Excuse me, where is the nearest post office?B:__________a.It’s on the left of the street.b.It’s on the right of the street.c.It’s across the street.d.It’s behind the bank.Answer: cExplanation: The question asks for the location of the nearest post office. The correct answer is “It’s across the street,” which is indicated by optionc.10.Listen to the following passage and answer the question.W: Hello, my name is Lisa. I’m from China. I’m here to study English.M: Hi, Lisa. It’s great to meet you. Where are you studying?W: I’m studying at XYZ English School.M: That’s a nice school. Do you like it?W: Yes, I do. The teachers are very nice, and the students are friendly.Question: Where is Lisa studying?a.ABC English Schoolb.XYZ English Schoolc.PQR English Schoold.STU English SchoolAnswer: bExplanation: The passage clearly states that Lisa is studying at XYZ English School, which is indicated by option b.11.You are listening to a conversation between two students, Tom and Lucy. Listen carefully and choose the correct answer.A. Tom is reading a book.B. Lucy is watching a movie.C. They are both studying English.Answer: BExplanation: The convers ation goes like this: “Hi, Tom, what are you doing?” “Hi, Lucy. I’m watching a movie. How about you?” “I’m reading a book. It’s really interesting.” The correct answer is B because Lucy mentions that sheis watching a movie.12.Listen to a short dialogue between a teacher and a student about the weekend plans. Then, choose the correct answer.A. The student is planning to go to the park with friends.B. The student is planning to visit a museum.C. The student is planning to stay at home and do some homework.Answer: AExplanation: The dialogue goes like this: “Teacher: What are you planning to do this weekend, Tom?” “Student: I’m planning to go to the park with my friends. We’re going to play soccer and have a picnic.” The correct answer is A because the student mentions that they are planning to go to the park with friends.二、选择题(本大题有12小题,每小题2分,共24分)1、Choose the correct word to complete the sentence.The cat is very __________, so it always keeps its fur clean.A. lazyB. cleverC. tidyD. hungryAnswer: C. tidyExplanation: The sentence is about the cat’s habit of keeping its fur clean, which suggests it is well-groomed and organized. “Tidy” is the correct choice as it implies cleanliness and orderliness.2、Select the word that is spelled differently from the others.A. changeB. flameC. strangeD. changeableAnswer: D. changeableExplanation: All the other words (change, flame, strange) have a double ‘e’ at the end. “Changeable” is the only word in the list that does not follow this pattern, making it the correct answer.3、Choose the correct word to complete the sentence.The teacher asked the students to turn off their__________before the class began.A. computersB. booksC. pensD. bagsAnswer: A. computersExplanation: The correct answer is “computers” because it is common for students to turn off their electronic devices, such as computers, before starting a class. The other options, while relevant to school, are not typically things that would need to be turned off before the class begins.4、Read the sentence and choose the best word to fill in the blank.She was__________surprised to see her friend after all these years.A. reallyB. veryC. muchD. soAnswer: D. soExplanation: The correct answer is “so” because it is used to emphasize the degree of surprise. The phrase “so surprised” is a common way to express a very high level of surprise. The other options can also express surprise, but they do not carry the same emphasis as “so.”5.Choose the correct word to complete the sentence.The cat is sleeping on the ____________.A. chairB. floorC. deskAnswer: BExplanation: The correct answer is “floor” because it is common to find a cat sleeping on the floor. “Chair” and “desk” are less likely places fora cat to sleep.6.Select the sentence that correctly uses the past tense.A. I go to the park yesterday.B. He was walking to the store last night.C. We are eating dinner at the restaurant now.Answer: BExplanation: The correct sentence is “He was walking to the store last night” because it uses the past continuous tense, indicating an action that was happening in the past. “I go to the park yesterday” is incorrect because “go” should be “went” in the past tense. “We are eating dinner at the restaurant now” is in the pre sent continuous tense and does not match the requirement of using the past tense.7、Choose the correct word to complete the sentence.The cat is very ______; it always lands on its feet.A)braveB)carefulC)luckyAnswer: C) luckyExplanation: The sentence implies that the cat has a remarkable ability to land on its feet, which is often associated with luck rather than bravery or caution.8、Select the sentence that does not follow the correct subject-verb agreement.A)The children are playing outside.B)Each child has their own book.C)She and her friends are going to the park.Answer: B) Each child has their own book.Explanation: In English, the correct subject-verb agreement for the phrase“each child” is “has” (singular verb), but the sentence incorrectly uses “their,” which is plural. The correct sentence should be “Each child has his or her own book.”9.Choose the correct word to complete the sentence:The cat was sleeping on the mat when the___________dog barked loudly.A. tinyB. angryC. happyD. lazyAnswer: BExplanation: The correct answer is “angry” because it implies that the dog’s barking was caused by frustration or irritation, which is a common reason fora dog to bark loudly.10.Select the sentence that has the correct subject-verb agreement.A. The students is studying in the library.B. Each student are reading books.C. The children has finished their homework.D. Both of the boys are playing outside.Answer: DExplanation: The correct answer is “Both of the boys are playing outside” because “both” is a pronoun that requires a plural verb, and “are” is the correct plural form of the verb “to be.” The other options have incorrectsubject-verb agreement.11.Choose the correct word to complete the sentence.The teacher was (A) angry, (B) surprised, (C) excited about the students’ performance in the play.Answer: BExplanation: The sentence is about the teacher’s reaction to the students’ performance. “Surprised” is the most appropriate word to describe thete acher’s reaction, as the performance may have been unexpected.12.Select the word that does not belong in the following list.(A)umbrella (B) car (C) bicycle (D) airplane (E) phoneAnswer: EExplanation: All the other items in the list are means of transportation that can be used on land. A phone is not a form of transportation and is used for communication purposes, making it the odd one out.三、完型填空(10分)Directions: There are 10 blanks in the following passage. For each blank, there are four choices marked A, B, C, and D. Please choose the best one for each blank.My favorite animal is the bear. Bears are very strong and intelligent. They live in the forest and eat a lot of different things. Bears can be black, brown, or even white. They are mostly herbivores, but some can also eat meat.One day, a bear was walking in the forest. He saw a small stream of water.He was very thirsty and wanted to drink. However, the stream was surrounded by high rocks. The bear tried to jump over the rocks, but he couldn’t make i t. He looked around and saw a vine growing near the rocks. He thought, “Maybe I can use this vine to climb over the rocks.”1.My favorite animal is the bear. It is a(an)________animal.A. strong and intelligentB. weak and lazyC. cute and playfulD. fierce and dangerous2.Bears live in the ________.A. cityB. forestC. oceanD. desert3.Bears can be ________.A. only blackB. only brownC. only whiteD. black, brown, or white4.Bears are mostly ________.A. carnivoresB. herbivoresC. omnivoresD. birds5.The bear saw a________growing near the rocks.A. treeB. flowerC. vineD. rockAnswers:1.A2.B3.D4.B5.C四、阅读理解(26分)Reading ComprehensionRead the following passage and answer the questions that follow.In the small town of Willow Creek, there was once a magnificent old oak tree that stood tall and proud at the center of the town square. The tree was known to everyone as “Old oak.” It was not just a tree; it was a symbol of the town’s history and community spirit. The children would gather under its broad branches during summer afternoons, the elders would sit on its roots and share stories,and during festive occasions, it was the perfect backdrop for parades and concerts.One day, a storm came, and the old oak tree was struck by lightning. Its branches snapped and its trunk began to crack. The townspeople were heartbroken at the sight of their beloved tree. They decided to take action and raise funds to save the old oak. The whole community contributed, from the smallest child to the oldest resident, and soon enough, enough money was gathered to hire a team of experts to help.The team worked tirelessly to stabilize the tree. They reinforced the trunk with steel and replaced the snapped branches with new, healthy ones. The old oak tree was saved, and the community celebrated with a grand festival. From that day on, they took even better care of the tree, ensuring it would stand tall for generations to come.Questions:1.Why was the old oak tree in Willow Creek significant to the community?A) It was the town’s only tree.B) It was a symbol of the town’s history and community spirit.C) It was the home of a local squirrel family.D) It provided shade for the town’s animals.2.What happened to the old oak tree during the storm?A) It was completely destroyed.B) Its branches snapped and its trunk cracked.C) It fell over due to its age.D) It was uprooted by the wind.3.How did the community help save the old oak tree?A) They planted new trees around it.B) They offered prayers for its recovery.C) They raised funds and hired experts to repair it.D) They moved the tree to a safer location.Answers:1.B) It was a symbol of the town’s history and community spirit.2.B) Its branches snapped and its trunk cracked.3.C) They raised funds and hired experts to repair it.五、写作题(16分)Task: Write a short story about a day you remember well. Your story should include the following elements:•The date and the occasion of the event.•The setting and the atmosphere of the day.•Three key events that happened, and their impact on you.•How the day has affected you since then.Example:Title: The Day of the FairOn October 15th, 2021, I experienced one of the most memorable days of my life. It was the annual school fair, and the whole campus was buzzing withexcitement.Setting and Atmosphere:The morning was crisp, with a gentle breeze that carried the scent of autumn leaves. The fairground was decorated with colorful balloons and festive banners. The atmosphere was electric, with children laughing and parents cheering.Key Events:1.Winning the Raffle: I had entered the raffle with a small toy car, and to my surprise, my name was drawn! I was so thrilled that I could barely contain my excitement.2.Performing in the Talent Show: Later in the afternoon, I decided to participate in the talent show. I sang a song I had been practicing for weeks, and the crowd cheered me on. I felt a sense of pride and accomplishment that day.3.Visiting the Craft Booth: I spent some time at the craft booth, makinga friendship bracelet with a fellow classmate. It was a simple activity, but it made me realize the importance of friendship and sharing.Impact and Reflection:Since that day, I have carried the joy and sense of community with me. The fair taught me the value of hard work, the importance of having fun, and the joy of making memories with friends. Every time I look at the friendship bracelet I made, I am reminded of that wonderful day and the positive impact it had on my life.Analysis:This example follows the structure outlined in the task. It starts with a clearintroduction of the date and occasion, followed by a description of the setting and atmosphere. The three key events are detailed, showing their impact on the narrator. Finally, the story concludes with a reflection on how the day has affected the narrator’s life since then, providing a personal touch and closure to the story.。
小学英语(闽教版)第六册期中测试卷(含答案及听力材料)
小学英语(闽教版)第六册期中测试卷(满分100分完卷时间:40分钟)听力部分(40分)6一、听音,写出你所听到的字母(包括大小写)。
(4分)1. 2. 3. 4.二、听音,补全单词。
(10分)1.sh_w ( )2. l_ss_n ( )3.c_ld ( ) 4 . f_m_us ( )5.h__ __r ( )三、听音,选择你所听到的单词的序号。
(6分)()1. A. homework B. housework ()2. A. hope B. help ()3. A. bright B. light ()4. A. catch B. watch ()5. A. ball B. fall ()6. A. heavy B. healthy 四、听音,在正确的图下打“√”。
(8分)五、听音,选择正确的答案。
(6分)( ) 1. Which Skirt?A: The short one . B: The long one.( ) 2. Is he in his bedroom?A. No, he isn’t.B. Yes, he is.( ) 3. Did you go anywhere in winter?A. Yes, I did. No, I didn’t.六、听音,在你所听到的句子前打勾,并在线上写出中文意思。
(4分)1. ( ) My birthday is February 3th.2. ( ) I can’t ride my bike. I’m very sad. ( ) My birthday is August 2nd. ( ) A dog is coming. I’m very scared. ______________________________ ________________________________3. ( ) We’re going for a spring outing.4. ( ) When is Easter?( ) We’re going to climb a mountain. ( ) When is Christmas?______________________________ ______________________________笔试部分(60分)一、按照要求写出所缺的字母(包括大小写)。
小学六年级上册第6次英语第六单元期中试卷(答案和解释)
小学六年级上册英语第六单元期中试卷(答案和解释)英语试题一、综合题(本题有50小题,每小题2分,共100分.每小题不选、错误,均不给分)1.Which of these animals is famous for its ability to jump?A. DogB. ElephantC. FrogD. Fish2.What is the opposite of "big"?A. SmallB. TallC. HeavyD. Fast3.What do we use to cut vegetables?A. KnifeB. PlateC. SpoonD. Fork4.In the morning, I always eat breakfast before going to school. My favorite breakfast is __ with butter and jam. I also drink a glass of orange juice. After breakfast, I pack my school bag with books, my lunchbox, and my __. Then, I put on my shoes and leave for school.5.Which of these is a body part?A. TableB. ChairC. ArmD. Spoon6.What is the opposite of "fast"?A. SlowB. TallC. HotD. Big7.What do you call a baby cat?A. PuppyB. CubC. KittenD. Calf8.Which animal is known for its long neck?A. ElephantB. GiraffeC. LionD. Tiger9.Which of these is a food?A. BreadB. ChairC. TableD. Spoon10.What is the opposite of "fast"?A. SlowB. BigC. TallD. Hot11.Which of these is a school subject?A. soccerB. scienceC. paintingD. running12.Tom and his family went to the __ last weekend. They saw many animals such as lions, __, and __. Tom’s favorite animal was the __ because it looked very cute. After visiting the zoo, they went to a nearby __ for lunch.13.He is __ in the class.A. the tallestB. tallestC. more tallD. tall14.What do we wear on our feet?A. HatB. GlovesC. ShoesD. Scarf15.I _______ (never/see) a movie like that before! The story _______ (be) so exciting, and the special effects _______ (be) amazing. After the movie, I _______ (talk) to my friends about it. We all _______ (agree) it _______ (be) the best movie of the year.16.What is the opposite of "happy"?A. SadB. TallC. BigD. Strong17.Which one is a vegetable?A. AppleB. CarrotC. PearD. Banana18.Which of these animals can fly?A. ElephantB. BirdC. FishD. Dog19.What do you use to cut paper?A. PenB. ScissorsC. SpoonD. Knife20.Which is the correct plural form of "child"?A. ChildsB. ChildrenC. ChilderD. Childes21.If you want to build a snowman, which of these materials would you need?A. SandB. SnowC. LeavesD. Grass22.How many eyes do most people have?A. 1B. 2C. 3D. 423.Which one is the correct spelling?A. CatB. CattC. CahtD. Caat24.Which one is a number?A. SevenB. SpoonC. ChairD. Table25.Which of these is a vegetable?A. BananaB. CarrotC. PearD. Apple26.What is the main color of grass?A. RedB. GreenC. BlueD. Yellow27.I ______ (like) to play video games with my brother. Every weekend, we ______ (play) together for a few hours. Last Saturday, we ______ (play) a new game, and it______ (be) very exciting. After that, we ______ (watch) a movie.28.What do we use to cut food?A. SpoonB. ForkC. KnifeD. Plate29.Sarah always ______ (help) her mom with housework. Yesterday, she ______ (clean) the kitchen and ______ (wash) the dishes. After that, she ______ (relax) and______ (watch) TV.30.Which one is a vehicle?A. AppleB. BikeC. CatD. Book31.What do we use to eat pasta?A. SpoonB. ForkC. KnifeD. Plate32.He ________ a good student.A. isB. areC. amD. be33.I’m excited because I’m going to visit my __ this weekend. They live in a big __. My cousin and I will play video games, and then we will go outside to play __. I think we will have __ fun! I can’t wait to see them again. Afterward, we’ll have a nice meal with __.34.Which of these is a color?A. AppleB. ChairC. BlueD. Tree35.What shape has 4 equal sides?A. CircleB. SquareC. TriangleD. Rectangle36.Which of these is a shape?A. CircleB. TableC. DogD. Car37.Which of these is a place where we can swim?A. LibraryB. HospitalC. PoolD. School38.Which of these is a food?A. ChairB. AppleC. SpoonD. Plate39.Which of these is used for drawing?A. EraserB. PencilC. PlateD. Fork40.In the morning, I always wake up at __ o’clock. I get out of bed, brush my __, and then have __ for breakfast. After breakfast, I put on my __ and go to school. At school, we study __ and do some fun activities.41.I have a _______ dog.A. smallB. catC. happyD. teacher42.What do you say when you meet someone for the first time?A. GoodbyeB. Thank youC. HelloD. Sorry43.Which one is a toy?A. ChairB. TableC. DollD. Plate44.What color is a banana?A. GreenB. RedC. YellowD. Blue45.Which of these is a body part?A. ArmB. ChairC. SpoonD. Plate46.Which sentence is a correct question?A. Where is your name?B. What is your name?C. Why is your name?D. Who is your name?47.In the winter, I love to go ice skating with my friends. The ice rink is very slippery, but we have a lot of fun. Sometimes, we play ice hockey, and other times, we just skate around and race each other.48.I ________ to school by bus.A. goB. goesC. goingD. gone49.Which of these is a shape?A. SquareB. TableC. ChairD. Car50.What do we use to clean our teeth?A. ToothbrushB. KnifeC. SpoonD. Fork(答案及解释)。
牛津译林版英语小学六年级上学期期中试题及解答参考(2024-2025学年)
2024-2025学年牛津译林版英语小学六年级上学期期中自测试题及解答参考一、听力部分(本大题有12小题,每小题2分,共24分)1、What does the boy want for breakfast?A) Toast and juiceB) Cereal and milkC) Eggs and baconAnswer: B) Cereal and milkExplanation: In the dialogue, the boy asks his mother if there’s any cereal left because he forgot to eat it yesterday. His mother confirms that there is still some cereal and fresh milk in the fridge.2、Where did the girl lose her backpack?A) At the libraryB) On the school busC) In the parkAnswer: A) At the libraryExplanation: During the conversation, the girl mentions that she realized her backpack was missing after she returned from studying at the library. She thought she had placed it under the table there.3、Listen to the dialogue between two friends and answer the question.A. What are the friends doing?B. Where are they going?C. What time is it?Answer: B. Where are they going?Explanation: Listen carefully to the dialogue. You will hear one of the friends say, “Let’s go to the library after school.” This indicates that they are discussing where to go after school.4、Listen to the passage about animals and choose the correct answer to the question.What is the main idea of the passage?A. The passage is about the different types of animals in the forest.B. The passage talks about how animals communicate with each other.C. The passage describes the habitat of various animals.Answer: C. The passage describes the habitat of various animals.Explanation: Listen to the passage carefully. The speaker talks about different animals and their living environments, such as forests, mountains, and oceans. The main idea of the passage is to describe the habitats of various animals.5、Listen to the conversation and choose the correct answer. (Listen once)A. They are going to the cinema.B. They are planning a picnic.C. They are discussing homework.Answer: B. They are planning a picnic.Explanation: In the conversation, you hear two friends talking about buying food and drinks and checking the weather forecast for tomorrow. This indicates they are making plans for a picnic.6、Listen to the short passage and decide if the following statement is true or false. (Listen twice)The school’s annual sports day wil l be held next Friday.Answer: True.Explanation: The passage mentions that all students should wear their sports uniforms and gather on the playground by 8 AM next Friday for the start of the school’s annual sports day event.7.Listen to the conversation between two students, Tom and Alice, discussing their weekend plans. Choose the correct answer from the options A, B, C, or D.A. Tom and Alice are going to the beach this weekend.B. Tom and Alice are planning to stay home and watch movies.C. Tom and Alice are going to visit their grandparents.D. Tom and Alice are preparing for a science fair.Answer: CExplanation: In the conversation, Tom mentions that they are visiting their grandparents in the countryside, which indicates option C is the correct answer.8.Listen to a short passage about a famous author, J.K. Rowling. Answer the following question based on the passage.Question: What inspired J.K. Rowling to write the Harry Potter series?A. Her experience of living in a small village.B. Her love for animals and nature.C. Her own childhood memories.D. Her fascination with the supernatural.Answer: CExplanation: In the passage, it is mentioned that J.K. Rowling drew inspiration from her own childhood memories to create the magical world of Harry Potter, making option C the correct answer.9、Listen to the dialogue and choose the correct answer.Dialogue:M: Excuse me, could you tell me where the nearest bookstore is?W: Of course. You need to go straight down this street until you reach the traffic lights. Turn left there, and the bookstore is on your right, next to the coffee shop.Question:Where is the bookstore according to the woman?A. Next to the post office.B. On the left of the coffee shop.C. On the right of the coffee shop.Answer: C. On the right of the coffee shop.Explanation: The woman in the dialogue clearly states that after turningleft at the traffic lights, the bookstore is on the right side, next to the coffee shop.10、Listen to the conversation and choose the correct answer.Dialogue:W: What do you think about the new science teacher?M: Well, his classes are always full of interesting experiments, but sometimes I wish he would slow down a bit because he covers too much material too quickly.Question:What does the bo y think about the new science teacher’s classes?A. They are too boring.B. They are too fast-paced.C. They are too slow.Answer: B. They are too fast-paced.Explanation: The boy mentions that while the classes are interesting due to the experiments, he feels that the teacher covers too much material too quickly, indicating that the pace of the class is too fast for his liking.11.Listen to the conversation between two students about their weekend plans. Choose the best answer.A. They are going to the beach.B. They are planning to visit their grandparents.C. They are going to a movie.Answer: BExplanation: The conversation mentions that they haven’t seen their grandparents for a while and are planning to visit them this weekend, which indicates that option B is correct.12.Listen to the following dialogue and answer the question.Why does the bo y want to borrow the girl’s bike?A. He has a flat tire.B. His mother forgot to buy him a new bike.C. He wants to ride it to school because it’s sunny.Answer: CExplanation: The boy says, “I really need your bike today because it’s such a nice day outsi de. I want to ride it to school instead of walking.” This indicates that option C is the correct answer.二、选择题(本大题有12小题,每小题2分,共24分)1、Which of the following words has a different pronunciation for the underlined part?A)heavyB)horseC)hourD)houseAnswer: C) hourExplanation:In options A, B, and D, the ‘h’ at the beginning of the word is pronounced, but in “hour,” the ‘h’ is silent, making its pronunciationdifferent from the other choices.2、What does the phrase “break the ice” mean?A)To start a conversation in a friendly wayB)To interrupt someone when they are speakingC)To end a meeting or gatheringD)To solve a difficult problemAnswer: A) To start a conversation in a friendly wayExplanation:The phrase “break the ice” is used to describe the action of doing or saying something to make people feel more relaxed and comfortable, especially when they meet for the first time or if there’s an awkward silence.3、What is the correct plural form of the word “child”?A. childsB. childrenC. childesD. childenAnswer: B. childrenExplanation: The plural form of “child” is “children.” The other options are incorrect spellings.4、Which of the following sentences uses the past continuous tense correctly?A. He was walking to the park when it started to rain.B. I am eating ice cream right now.C. She has been sleeping for 8 hours.D. They went to the movies yesterday.Answer: A. He was walking to the park when it started to rain.Explanation: The past continuous tense is used to describe an action that was ongoing at a specific time in the past. Option A correctly uses the past continuous tense (“was walking”) to indicate an action that was happening when another past action occurred (“started to rain”). Options B and D use the simple past tense, and option C uses the present perfect continuous tense.5、Choose the correct word to complete the sentence.The cat was___________on the windowsill, looking out at the birds.A)layingB)lyingC)lying downD)layAnswer: B) lyingExplanation: The correct word here is “lying,” which means to be in a horizontal position, often relaxed. The cat is simply in a horizontal position on the windowsill, not sleeping or doing anything else. “Laying” (A) is incorrect because it implies the act of placing something down, and “lying down”(C) is too specific as it suggests the cat is in a resting position, which is not necessary to describe the action of simply being horizontal. “Lay” (D)is the past tense of “lie” and does not fit the present continuous context of the sentence.6、Complete the sentence with the correct form of the verb in parentheses. The children___________(be) very excited about the upcoming school trip.A)areB)isC)wereD)beAnswer: A) areExplanation: The correct form of the verb is “are,” as it matches the plural subject “The children.” The sentence describes a present state of excitement, so the present simple tense is appropriate. “Is” (B) is incorrect because it is the singular form of the verb and does not match the plural subject. “Were” (C) is the past tense and does not fit the present context. “Be” (D) is the base form of the verb and does not agree with the plural subject in terms of number.7、Choose the correct word to complete the sentence.The cat is_______(sleeping/awake) on the mat.A. sleepingB. awakeAnswer: AExplanation: The sentence talks about a cat on a mat, which implies it is resting. Therefore, “sleeping” is the correct word to complete the sentence.8、Select the word that is different from the others in the group.A. umbrellaB. appleC. pencilD. watchAnswer: BExplanation: The words “umbrella,” “pencil,” and “watch” are all objects that people use. “Apple,” on the other hand, is a type of fruit, which makes it different from the other words in the group.9.Choose the word that best completes the sentence.The teacher praised the students for their__________behavior in the classroom.A)aggressiveB)cooperativeC)passiveD)rebelliousAnswer: B) cooperativeExplanation: The correct answer is “cooperative” because it implies that the students were working well together and showing a willingness to help and collaborate with others, which is positive behavior in a classroom setting. The other options (“aggressive,” “passive,” “rebellious”) suggest behavior that is not typically praiseworthy in an educational context.10.Select the sentence that has the correct use of the present perfect tense.A)She has gone to the store already.B)He is gone to the store since yesterday.C)They have been gone to the movies for two hours.D)We had gone to the park before the rain started.Answer: A) She has gone to the store already.Explanation: The present perfect tense is used to describe an action that started in the past and continues to the present or an action that has a present result. Option A correctly uses the present perfect tense to indicate that the action of going to the store has happened in the past and might still be ongoing. Options B, C, and D are incorrect because they use the past perfect, present perfect continuous, or past perfect continuous tenses, which are not appropriate in this context.11.Choose the correct word to complete the sentence.The teacher asked us to hand in our homework ____________.A. soonB. quicklyC. earlyD. latelyAnswer: B. quicklyExplanation: The word “quickly” is the correct choice because it means to do something with speed or in a short time, which is suitable for the context of handing in homework. “Soon” means at a later time, “early” means before the scheduled time, and “lately” means in recent times, none of which fit the context as well as “quickly”.12.Select the word that does not belong in the following list.1.apple2.banana3.orange4.pear5.grape6.peach7.pearAnswer: 7. pearExplanation: The word “pear” appears twice in the list, which makes it the odd one out. All the other items are unique fruits, while “pear” is repeated.三、完型填空(10分)Title: Oxford Treehouse Edition English Grade 6A Mid-term ExamSection 3: Cloze TestDirections: Read the passage carefully and choose the best answer for each blank.The following story is about a young girl named Lily who loves reading. She often goes to the library to borrow books.Lily was very excited because she had received a special gift from her grandmother. It was a beautiful 1(bookshelf) for her room. She had always wanted a bookshelf to put all her favorite books on.One evening, Lily decided to 2 (organize) her new bookshelf. She started by 3(arrange) her books by genre. She put all her 4(science) books on one shelfand her 5 (adventure) books on another.As she was arranging her books, Lily realized that she had many books to read. She couldn’t wait to dive into her new collection and explore new worlds.1.a. shelf2.a. arrange3.a. arrange4.a. science5.a. adventureAnswer: 1. a. shelf 2. a. arrange 3. a. arrange 4. a. science 5. a. adventure 四、阅读理解(26分)Reading ComprehensionPassage:The British Museum is located in London, England. It is one of the largest and most famous museums in the world. The museum was founded in 1753 and opened to the public in 1759. The British Museum houses a vast collection of art and artifacts from all over the world. It includes items from ancient civilizations like Egypt, Greece, and Rome, as well as from the Middle East and Asia.One of the most famous items in the museum is the Rosetta Stone. The Rosetta Stone is a granodiorite stele inscribed with a decree issued in 196 BC on behalf of King Ptolemy V. The stone is famous because it contains a decree written in three scripts: hieroglyphic, Demotic, and Greek. The discovery of the RosettaStone in 1799 by French soldiers during the Napoleonic Wars helped decipher Egyptian hieroglyphs and opened the way for understanding ancient Egyptian history.Another significant item is the Elgin Marbles, which were taken from the Parthenon in Athens, Greece, by Lord Elgin in the early 19th century. The Parthenon was a temple dedicated to the goddess Athena. The Elgin Marbles include the frieze, metopes, and pediments from the Parthenon. These marble sculptures depict various gods, goddesses, and heroes from Greek mythology.The British Museum also has a large collection of coins, medals, and coins from ancient civilizations. The coins and medals collection provides valuable insights into the economic and political history of various civilizations.Questions:1.When was the British Museum founded?A) 1753B) 1759C) 196 BCD) 17992.What is the significance of the Rosetta Stone?A) It is a granodiorite stele inscribed with a decree.B) It contains a decree written in three scripts.C) It depicts gods, goddesses, and heroes from Greek mythology.D) It was taken from the Parthenon in Athens.3.Which of the following is true about the Elgin Marbles?A) They are a collection of coins and medals.B) They include the frieze, metopes, and pediments from the Parthenon.C) They are from ancient Egyptian history.D) They were taken from the British Museum.Answers:1.A) 17532.B) It contains a decree written in three scripts.3.B) They include the frieze, metopes, and pediments from the Parthenon.五、写作题(16分)Title: My Ideal SchoolWriting Prompt: Imagine a perfect school where everything is just right. Describe your ideal school, including the facilities, teachers, and students. Why do you think such a school would be beneficial for students?Example:My Ideal SchoolIn my ideal school, everything is designed to foster a positive and enriching learning environment. The campus is situated in a serene location, surrounded by lush greenery and a sparkling lake. The buildings are modern and well-equipped, with spacious classrooms, well-stocked libraries, and advanced technology labs.The teachers in my ideal school are highly qualified and passionate about their profession. They are patient, approachable, and dedicated to helping eachstudent reach their full potential. They use innovative teaching methods and encourage students to think critically and creatively. The teachers also organize various extracurricular activities, such as sports, music, and art clubs, to promote holistic development.The students in my ideal school come from diverse backgrounds, but they all share a common goal of learning and growing. They are respectful, cooperative, and supportive of each other. The school culture is inclusive and encourages students to embrace their unique talents and interests.One of the key features of my ideal school is the emphasis on sustainability. The campus is powered by renewable energy sources, and students are encouraged to participate in environmental conservation projects. This not only teaches them about the importance of taking care of our planet but also instills in them a sense of responsibility.I believe such a school would be beneficial for students because it provides them with a well-rounded education that caters to their academic, social, and emotional needs. The supportive environment allows students to thrive and develop into well-rounded individuals who are ready to face the challenges of the future.Analysis:In this example, the student has clearly described their ideal school, covering various aspects such as facilities, teachers, and students. The example demonstrates a strong understanding of the topic and effectively communicatesthe benefits of an ideal school environment. The writer uses vivid and descriptive language to create a picture of the perfect school, and they provide clear reasons for why such a school would be beneficial for students. The example also shows an understanding of the importance of sustainability in education.。
小学六年级上册第6次英语第六单元期中试卷(答案和解释)
小学六年级上册英语第六单元期中试卷(答案和解释)英语试题一、综合题(本题有50小题,每小题2分,共100分.每小题不选、错误,均不给分)1.Which of these is a type of tree?A. PineB. AppleC. PearD. Banana2.Which one is a vegetable?A. BananaB. CarrotC. MangoD. Apple3.Which of these animals is a pet?A. LionB. DogC. ElephantD. Kangaroo4.This is my school. It is very __ and there are many __. In the morning, we have __ classes. After that, we go to the __ to have lunch. In the afternoon, we usually have __ or __. My friends and I like playing __ after school.5.What do we drink?A. BreadB. MilkC. ChairD. Bed6.Which is the correct way to say "谢谢" in English?A. PleaseB. SorryC. Thank youD. Welcome7.Which of these is the tallest animal?A. ElephantB. GiraffeC. TigerD. Lion8.I __________ (play) the piano every evening. My sister __________ (play) the guitar. We __________ (practice) together for one hour every day. Last week, we __________ (perform) at a school concert. It __________ (be) a great experience.9.There are many animals at the ______. I like to see the ______ because they are so big and strong. I also like the ______, which are very cute. After visiting the animals, we have a ______ and talk about what we saw. It’s always a great experience!10.Which one is a type of fruit?A. OrangeB. OnionC. CarrotD. Potato11.Which of these is a sport?A. paintingB. soccerC. runningD. singing12.Which of these is an animal?A. TreeB. LionC. SpoonD. Platest summer, I ______ (go) to the beach with my family. We ______ (swim) in the sea and ______ (build) sandcastles. My little brother ______ (enjoy) playing with the sand, but I ______ (like) swimming more. After the beach, we ______ (have) ice cream, and it ______ (be) delicious.14.What do we use to measure time?A. PencilB. WatchC. PlateD. Chair15.Which one is a color?A. MilkB. RedC. TableD. Dog16.Sarah loves going to school because she enjoys learning new things. She always starts her day by packing her __________ with books, notebooks, and a __________. After she arrives at school, she meets her friends, and they go to the __________ to play before class begins. In class, Sarah listens carefully to the teacher and takes notes with her __________. After school, Sarah usually goes to the library to borrow some__________, which she enjoys reading at home.17.This morning, I ______ (take) the bus to school. On the way, I ______ (see) a cat sitting on a fence. The cat ______ (look) very cute, so I ______ (stop) to pet it. After that, I ______ (continue) my journey to school. When I ______ (arrive), my friends ______ (wait) for me at the school gate.18.I woke up early this morning because I had to go to school. After brushing my __ and washing my __, I ate some __ for breakfast. I packed my __ with my books and went to school. During class, we learned about the __ and how it helps us grow. I love learning new things at school!19.Tim likes to go to the __________ to buy food. His family needs to buy bread,__________, __________, and some __________ for the week. Tim’s mother loves tocook __________ for dinner. Tim also likes to help her __________ the vegetables. After shopping, they go home and start cooking.20.At the zoo, Lily saw many __________ (1). There were some __________ (2) in the big cages, and she also saw some __________ (3) running around. Her favorite part of the zoo was the __________ (4) section, where she watched the elephants play in the __________ (5). After visiting the animals, Lily and her family went to the __________ (6) to eat lunch.21.Which of these is a place where we live?A. SchoolB. OfficeC. HouseD. Park22.He ______ a teacher.A. amB. isC. areD. be23.Which of these is a type of fruit?A. AppleB. CarrotC. LettuceD. Potato24.In the summer, we __________ (go) to the beach. We __________ (play) in the sand and __________ (swim) in the sea. My brother __________ (build) a sandcastle, and I __________ (collect) seashells. We __________ (eat) ice cream and __________ (have) a picnic on the beach. It __________ (be) so much fun. After the beach, we__________ (take) a bus to the park and __________ (stay) there until sunset.25.Mark is at school and ready to start his math homework. He opens his __________ and takes out a pencil. The first question asks him to __________ the numbers and write the answer on the line. Mark knows the answer is __________, so he writes it down confidently. After finishing the math homework, he starts his __________ homework, which is a little more difficult.26.Which of these is a body part?A. SpoonB. HandC. PlateD. Chair27.I enjoy __________ because it is very __________. I usually go there with my__________. We like to __________ in the __________ and watch the __________. Sometimes, we also play __________ in the __________. I love __________ because it makes me feel very __________.28.In the evening, I usually ______ (do) my homework and then ______ (watch) TV for a while. Last night, however, I ______ (read) a book instead of watching TV. The book ______ (be) about a detective who ______ (solve) mysteries. It ______ (be) very interesting, and I ______ (finish) it in one night.29.Lily loves to cook with her mom. Today, they are making a delicious __________. Lily helps by chopping some __________ and adding them to the pot. Her mom adds some __________ to make the soup taste good. After the soup is ready, they serve it with __________. Lily’s dad says it is the best soup he has ever tasted.st weekend, I went to the park with my family. We arrived at the park at 10:00 in the morning. The weather was __________ and sunny, so it was perfect for outdoor activities. I played __________ with my little brother and we __________ a big ball. After that, we went to the lake and saw some __________ swimming. My parents enjoyed walking around, and we all __________ lots of photos. In the afternoon, we had a picnic near the __________. We ate sandwiches, fruits, and __________. It was a__________ day, and we all had a great time!31.Which of these is a vehicle?A. CarB. TableC. ChairD. Dog32.Tommy loves to draw pictures. His favorite things to draw are __, __, and __. Yesterday, he drew a big __ on his notebook. His little sister also likes drawing and drewa picture of a __. Tommy is going to frame his picture and hang it on the wall.33.What do you use to cut paper?A. PenB. ScissorsC. RulerD. Eraser34.Which of these is a fruit?A. PotatoB. BananaC. CarrotD. Tomato35.Which of these is used to drink?A. ForkB. CupC. KnifeD. Plate36.Which of these is an insect?A. BirdB. ButterflyC. FishD. Lion37.Which of these is a time of day?A. JanuaryB. SummerC. MorningD. Blue38.What do we use to eat pasta?A. KnifeB. ForkC. SpoonD. Plate39.When I __________ (be) a child, I __________ (like) to play outside with my friends. We __________ (ride) bicycles and __________ (play) hideandseek. I__________ (remember) those days as the best time of my childhood.st weekend, my family and I __________ (1) to the zoo. It __________ (2) a very sunny day, and we __________ (3) many animals. The lions __________ (4) sleeping, but the monkeys __________ (5) playing in the trees. We __________ (6) pictures of all the animals and __________ (7) a great time.41.Which of the following sentences is correct?A. She go to school every day.B. She goes to school every day.C. She going to school every day.D. She gone to school every day.42.Which of these animals has feathers?A. FishB. LionC. BirdD. Elephant43.I ______ (take) a shower every morning before school. After I ______ (finish) my shower, I ______ (get) dressed and ______ (eat) breakfast. This morning, I ______ (eat) pancakes with syrup. They ______ (be) delicious, and I ______ (feel) full after breakfast.44.Which of these is a means of transportation?A. CarB. KnifeC. PlateD. Chair45.What color is an apple?A. RedB. BlueC. OrangeD. Green46.Which of these is used to play music?A. PianoB. PlateC. SpoonD. Chairst summer, my family and I ______ (go) to a beautiful beach for vacation. The weather ______ (be) perfect, with clear skies and a gentle breeze. Every day, we ______ (swim) in the sea, and my little brother ______ (build) sandcastles on the shore. We also ______ (collect) seashells and ______ (take) many pictures. At night, we ______ (eat) delicious seafood and ______ (watch) the sunset. It ______ (be) a wonderful trip, and we all ______ (have) a great time.48.Which of these is a wild animal?A. DogB. CowC. ElephantD. Tiger49.Which of these animals says "moo"?A. DogB. CatC. CowD. Bird50.Which season comes after winter?A. SpringB. SummerC. AutumnD. Winter again(答案及解释)。
小学六年级上册第六次英语第六单元期中试卷(答案和解释)
小学六年级上册英语第六单元期中试卷(答案和解释)英语试题一、综合题(本题有50小题,每小题2分,共100分.每小题不选、错误,均不给分)1.How many months are there in a year?A. 10B. 12C. 24D. 62.Which of these is used to tell time?A. ClockB. ShoeC. BookD. Plate3.Which of these is a pet?A. ElephantB. CatC. LionD. Giraffe4.My dog __________ (love) to play in the yard. He __________ (run) around and__________ (chase) after his favorite ball. Yesterday, he __________ (catch) the ball in midair! It __________ (be) amazing.5.Lily loves to read books. Today, she is reading a story about a __________ who goes on an adventure in the __________. The __________ finds a treasure map and follows the clues to find hidden gold. Along the way, the __________ faces many challenges but never gives up. Lily is very excited to find out what happens at the end of the story. After finishing the book, Lily puts it on the __________ in her room to read again later.6.Which one is a time of day?A. MorningB. DogC. SpoonD. Car7.David is going to the store to buy some __________. He needs to pick up some__________, __________, and __________ for his mom. After shopping, he goes to the cashier and pays for the items.8._______ they your friends?A. IsB. AreC. AmD. Be9.Jake is getting ready for a big __________ (1). His mom is baking a delicious__________ (2) in the kitchen. Jake is helping by setting the __________ (3) on the table. He also arranges the __________ (4) neatly and places the __________ (5) beside them. When everything is ready, Jake’s friend s will come and enjoy the party.10.Right now, I __________ (study) for my English test. I __________ (look) at my notes and __________ (practice) some vocabulary. I __________ (feel) confident because I __________ (study) hard.11.Which one is used to clean the floor?A. BroomB. PlateC. ChairD. Book12.Which one is a type of weather?A. SnowB. ChairC. BookD. Table13.What is the capital of England?A. LondonB. ParisC. TokyoD. New York14.Which of these is used to make a phone call?A. PhoneB. ChairC. PlateD. Spoon15.Which one is a fruit?A. DogB. AppleC. ChairD. Shirt16.In the __________ (1), it is very __________ (2). I like to wear my __________ (3) jacket when I go outside. Sometimes, it __________ (4) and I need to bring my__________ (5). My friends and I like to play __________ (6) together when it is cold. We also drink __________ (7) to keep warm.17.Which of the following is not a fruit?A. AppleB. OrangeC. CarrotD. Grape18.Which one of these is a season?A. WinterB. NoonC. OctoberD. Sunday19.Yesterday, I ______ (go) to the market with my mother. We ______ (buy) some fresh vegetables and fruit. After that, we ______ (stop) by a café and ______ (have) lunch together. I ______ (eat) a sandwich and my mother ______ (drink) a cup of tea. It ______ (be) a lovely day, and we ______ (enjoy) it very much.20.Which sentence is correct?A. He go to school every day.B. He goes to school every day.C. He going to school every day.D. He go to school everyday.21.At school, we have different subjects. In ______ class, we learn about countries and their people. In ______ class, we study numbers and solve problems. My favorite subject is ______ because I like to draw and create new things. I also enjoy ______, especially reading about animals and nature.22.It’s a __________ (1) morning. Tim and his sister, Kate, are going to the__________ (2) to buy some food. They need to buy bread, __________ (3), and some __________ (4). Tim likes to drink __________ (5), and Kate prefers __________ (6). After shopping, they return home and help their mom __________ (7) the kitchen.23.I’m preparing for my birthday party. I will invite my friends, and we will have lots of fun. We will play __, eat __, and listen to __. After the party, I will give everyone a small __ as a thank you.24.Which of these is a color?A. DogB. BlueC. ChairD. Book25.What do you drink from?A. PlateB. CupC. ShoeD. Hat26.What is the opposite of “young”?A. OldB. TallC. ShortD. Small27.I have a new bicycle. It is blue and white. Every weekend, I ride it to the park. I like riding my bicycle because it is fun and keeps me healthy. Sometimes, my friends come with me and we race each other.28.Which of these is a part of your body?A. LegB. ChairC. TableD. Bedst month, I ______ (start) learning how to play the guitar. I ______ (practice) every day for one hour. Yesterday, I ______ (learn) a new song.30.Which of these is a famous person?A. Walt DisneyB. SpoonC. PlateD. Chair31.Which of these is a transport?A. CarB. ChairC. TableD. Bed32.My school _______ (have) a big library. We _______ (borrow) books there every week. I _______ (like) reading books about animals and science. Last week, I _______ (borrow) a book about space.33.Which one is used to clean the floor?A. BroomB. KnifeC. PlateD. Spoon34.My friends and I __________ (1) to a park last weekend. The weather __________ (2) sunny and warm, perfect for playing outside. We __________ (3) a picnic on the grass and __________ (4) some sandwiches and fruits. After eating, we __________ (5) to play soccer. I __________ (6) the ball and __________ (7) a goal! We __________ (8) for a long time and __________ (9) very tired when it was time to go home.35.Which of the following is a vegetable?A. AppleB. CarrotC. BananaD. Orange36.I ______ (visit) my grandparents last weekend. They ______ (live) in the countryside, so we ______ (spend) the whole day there. We ______ (pick) apples from their garden, and my grandmother ______ (make) apple pie. I ______ (help) her bake the pie, and it ______ (taste) delicious.37.My name is Jack. I __________ (1) a student. I __________ (2) in a big city. I__________ (3) to school by bus every day. I __________ (4) math, but I __________ (5) science. My favorite food __________ (6) pizza. On weekends, I __________ (7) to the park with my friends. We __________ (8) soccer and __________ (9) ice cream after playing.38.Which one is used for writing?A. PlateB. PenC. PillowD. Shoe39.Which of these is used to brush your teeth?A. ToothbrushB. PenC. SpoonD. Towel40.Sarah is in the garden with her mom. They are planting some __________ in the soil. Sarah digs a hole with a __________, while her mom places the __________ in the ground. After they finish, they water the flowers with a __________. Sarah is very__________ because she loves plants.41.We are having a big family dinner tonight. My mom is making her special spaghetti with tomato __. My dad is grilling chicken, and I am setting the table. We are also havinga salad with __ and some bread. After dinner, we will have ice __ for dessert.42.Which one is not a fruit?A. PearB. PeachC. TomatoD. Cucumber43.Which one is a color?A. TableB. RedC. SpoonD. Chair44.How do you say "书" in English?A. BookB. PenC. TableD. Chair45.Which of these is a color?A. dogB. blueC. bookD. apple46.Tom and his brother are building a __________ (treehouse) in the backyard. They use __________ (wood), __________ (nails), and a __________ (hammer). Tom’s brother climbs up the __________ (ladder) to nail the wood to the __________ (branches). They hope to finish the treehouse before it starts __________ (raining).47.She is _______ than her brother.A. tallB. tallestC. tallerD. more tall48.This afternoon, I went to the library to borrow some books. I need them for my school project. The library is very quiet, and there are many books on different subjects. I borrowed three books: one about animals, one about space, and one about history. After borrowing the books, I sat down to read. I’m looking forward to finishing my project soon.49.What is the opposite of "fast"?A. SlowB. TallC. WideD. Heavy50.What do we use to drink water?A. PlateB. CupC. SpoonD. Knife(答案及解释)。
牛津版英语小学六年级上学期期中试题与参考答案(2024-2025学年)
2024-2025学年牛津版英语小学六年级上学期期中自测试题与参考答案一、听力部分(本大题有12小题,每小题2分,共24分)1、Listen to the following dialogue and choose the best answer to complete the sentence.A. How are you today?B. I’m fine, thank you.C. I have a test tomorrow.D. What’s your name?Answer: C.Explanation: The speaker is discussing an upcoming test, indicating that they have a test tomorrow.2、Listen to the question and then answer it.Question: What is the weather like in Oxford today?A. It’s sunny.B. It’s cloudy.C. It’s rainy.D. It’s windy.Answer: B.Explanation: The correct answer will be provided based on the actual weather conditions in Oxford on the day of the test. The question is asking for a specific detail that should be mentioned in the listening passage.3、Listen to the dialogue and choose the best answer to complete the sentence.A. The boy is playing soccer.B. The girl is reading a book.C. They are both doing their homework.Answer: BExplanation: In the dialogue, the speaker mentions that the girl is sitting at the table with a book in her hands, indicating that she is reading. There is no mention of soccer or homework.4、Listen to the following short passage and answer the question.What is the main idea of the passage?A. The importance of exercise for children.B. The benefits of eating a balanced diet.C. The advantages of playing outside during the day.Answer: CExplanation: The passage discusses how playing outside during the day can help children stay healthy, active, and improve their mood. Although exercise and a balanced diet are mentioned, the main focus is on the advantages of playing outside.5、Listen to the dialogue and answer the question.A. What is the weather like today?B. What are the students doing?C. How many books are there in the library?Dialogue:W: Good morning, everyone. Today we’re going to the library. The weather is sunny and quite warm.M: That sounds great! I can’t wait to read some books.W: There are many books available, but we need to remember to return them on time.Question: What are the students doing?Answer: B. What are the students doing?Explanation: The dialogue mentions that the students are going to the library to read books, which indicates they are engaged in an activity related to reading.6、Listen to the short passage and choose the best answer to the question.A. The boy is playing soccer.B. The girl is swimming.C. The family is having a picnic.Passage:Last weekend, the Smith family decided to go for a picnic. They packed a lot of food and drinks. While the adults were setting up the picnic blanket, the children played near the river. The boy was catching butterflies, and the girlwas swimming in the clear water.Question: What are the Smiths doing last weekend?Answer: C. The family is having a picnic.Explanation: The passage explicitly mentions that the Smith family is havinga picnic, which is the central activity described in the text.7.You are listening to a conversation between a teacher and a student.Teacher: “How was your weekend, Tom?”Student: “It wa s great, Mrs. Green. I went hiking with my family and we saw a lot of wildlife.”Question: What did Tom do over the weekend?A) He went to the movies.B) He went hiking with his family.C) He stayed home and read books.D) He visited his grandparents.Answer: B) He went hiking with his family.Explanation: The student explicitly mentions that he went hiking with his family, which is the correct answer.8.Listen to a short dialogue between two friends, Sarah and Alex, discussing their weekend plans.Sarah: “I’m thinking of going to the beach this weekend. Do you want to join me, Alex?”Alex: “That sounds fun! I’ve been wanting to go to the beach for a while. Whattime are you planning to leave?”Question: What is Alex’s response to Sarah’s invitation?A) She says she’s already made other plans.B) He agrees to go and asks about the departure time.C) He suggests they go to the amusement park instead.D) He declines and recommends a movie night.Answer: B) He agrees to go and asks about the departure time.Expl anation: Alex responds positively by saying “That sounds fun!” and then inquires about the departure time, indicating his willingness to join Sarah at the beach.9、Listen to the following conversation and choose the best answer to the question you hear.A. The children are going to the park.B. The children are going to the zoo.C. The children are going to the museum.Answer: BExplanation: The conversation mentions that the children are excited about going to the zoo, which indicates the correct answer.10、Listen to the short dialogue and complete the sentence with the missing word.W: What are you doing after school, Tom?M: I’m going to the________with my friends.A. libraryB. restaurantC. supermarketAnswer: AExplanation: The context suggests that Tom is going somewhere after school with friends, which is most likely a library where people often study or read. The other options, a restaurant and a supermarket, are less likely activities for after school with friends.11.Listen to the dialogue and choose the best answer to complete the following sentence.A. The children are playing soccer.B. The children are playing basketball.C. The children are playing tennis.Answer: BExplanation: In the dialogue, the children are mentioned to be playing a game of basketball, which corresponds to option B.12.Listen to the short passage about a book fair and answer the following question.Question: What is the main purpose of the book fair?A. To sell new books only.B. To sell both new and second-hand books.C. To provide information about the authors.Answer: BExplanation: The passage states that the book fair will have a variety of books for sale, including new and second-hand books, which aligns with option B.二、选择题(本大题有12小题,每小题2分,共24分)1、Choose the correct form of the verb to fill in the blank:The cat_______(is / are) sleeping on the windowsill.Answer: isExplanation: The subject “the cat” is singular, so we use the singular form of the verb “to be,” which is “is.”2、Select the correctly punctuated sentence:A. It’s raining today, we can’t go to the park.B. It’s raining today we can’t go to the park.C. It’s raining today; we can’t go to the park.D. Its raining today, we can’t go to the park.Answer: CExplanation: The corr ect choice is “It’s raining today; we can’t go to the park.” This sentence uses a semicolon to separate two independent clauses that are closely related in meaning. Option A incorrectly uses a comma before the conjunction when connecting two independent clauses. Option B lacks punctuation between the clauses. Option D has an incorrect apostrophe usage(“Its” instead of “It’s”) and also lacks necessary punctuation.3、The sentence “The cat is sleeping under the tree” can be transformed into a question using “What” as the question word. Which of the following is the correct question form?A. What is the cat sleeping?B. What is the cat sleeping under?C. The cat is sleeping what?D. The cat is sleeping where?Answer: BExplanation: The correct question form using “What” to ask about the location where the cat is sleeping would be “What is the cat sleeping under?” This question maintains the original meaning while asking for the specific location.4、Choose the word that best completes the sentence.The dog is very __________, and it always keeps us company when we are home alone.A. braveB. lazyC. happyD. helpfulAnswer: DExplanation: The word “helpful” best fits the context of the sentence,suggesting that the dog provides comfort or support when its owners are home alone. The other options do not necessarily convey the same sense of companionship or utility.5、Which of the following words is a synonym for “brave”?A)fearfulB)timidC)courageousD)hesitantAnswer: C) courageousExplanation: The word “courageous” means showing or having courage; brave. It is a synonym for “brave.” The other options are antonyms, as they describe someone who is not brave: “fearful” and “timid” mean being afraid, while “hesitant” suggests a lack of confidence or reluctance to do something.6、Choose the correct preposition to complete the sentence: She will arrive___6 o’clock.A)inB)onC)atD)byAnswer: C) atExplanation: The correct preposition to use with a specific time (such as 6 o’clock) is “at.” We say “at 6 o’clock” when we want to indicate the exact time. “In” is used with parts of the day (in the morning, in theafternoon), “on” is used with days and dates (on Monday, on Christmas Day), and “by” is used to show the deadline (by 6 o’clock, meaning no later than6 o’clock).7、The sentence “The cat sat on the mat” has been changed into the past continuous tense. Which of the following is the correct past continuous form of the verb “sat”?A)satB)was sittingC)had satD)would sitAnswer: B) was sittingExplanation: The past continuous tense is formed with “was/were” + the present participle (base form of the verb + -ing). In this case, the correct form for “sit” is “was sitting.”8、Choose the word that best completes the sentence.If it __________, we would have to postpone the picnic.A)rainsB)will rainC)rainedD)has rainedAnswer: A) rainsExplanation: The sentence describes a conditional situation, which is a present condition resulting in a future result. The correct form of the verbto use here is the simple present tense to express a general truth or a habitual action. Therefore, “rains” is the correct choice.9、Which of the following sentences is correct?A)She enjoy playing football after school.B)They enjoys playing football after school.C)He enjoys playing football after school.D)We enjoy play football after school.Answer: C) He enjoys playing football after school.Explanation:The correct form requires the subject ‘He’ to be followed by the verb ‘enjoys’ because ‘he’ is singular, an d the verb should agree with the subject in person and number. Also, the gerund ‘playing’ is used correctly as the object of the verb ‘enjoys.’10、If you want to ask someone if they can swim, which of the following would you say?A)Can you swimming?B)Do you can swim?C)Are you able to swim?D)You can swim?Answer: C) Are you able to swim?Explanation:To inquire about someone’s ability to do something, you can use the phrase ‘Are you able to…’ followed by the base form of the verb. Option A is incorrect because it mixes the modal verb ‘can’ with the gerund‘swimming.’ Option B incorrectly uses two modal verbs together. Option D isa statement rather than a question.11.Which of the following sentences uses the correct form of the past simple tense?A. He was eating breakfast when the doorbell rang.B. She had finished her homework before she left for school.C. They were watching TV while we were having dinner.Answer: AExplanation: The correct form of the past simple tense is used in sentence A. The past simple tense is used to describe actions that happened in the past and are not happening now. In sentence A, “was eating” and “rang” are in the past simple tense.12.Choose the correct word to complete the sentence.The cat_______(A) sit on the mat.Answer: AExplanation: The correct word to complete the sentence is “sat.” The past simple tense is needed here, as the sentence is describing an action that happened in the past. “Sat” is the past simple form of the verb “to sit.”三、完型填空(10分)Sure, I can create a cloze test question suitable for an English exam for sixth-grade students using Oxford English standards. Here’s an example with five blanks, including the answers:Cloze Test (Complete the passage with the correct words.)Read the following passage and fill in the blanks with the most appropriate word from the options given below. There might be extra words that you will not need to use.Passage:Last summer holiday, my family and I went on a trip to a beautiful seaside town called Marlow. The weather was perfect, 1 sunny but not too hot. We stayed in a cozy cottage near the beach where we could hear the waves at night. Every morning, we would go for a walk along the shore and collect 2 such as pretty pebbles and seashells. One day, we even saw some dolphins swimming close to the coast. It was such an 3sight that we couldn’t help but cheer. For dinner, we often had fresh seafood at a local restaurant that served delicious meals. The best part of our vacation, however, was when we took a boat trip to explore the nearby islands. We brought our picnic 4 and had lunch on a deserted beach. It was truly a memorable holiday, and I hope that we can visit Marlow again next year and make more 5 there.Options:A. always / souvenirs / exciting / basket / memoriesB. often / gifts / thrilling / box / souvenirsC. usually / shells / amazing / hamper / momentsD. frequently / trinkets / astonishing / sack / experiencesAnswers:1.A. always2.A. souvenirs3.A. exciting4.A. basket5.A. memoriesThis passage is designed to test vocabulary knowledge, comprehension skills, and the ability to use context clues to determine the correct word choice.四、阅读理解(26分)Reading ComprehensionPassage:The ancient city of Pompeii is one of the most famous archaeological sites in the world. Located in the Campania region of Italy, Pompeii was a bustling Roman city until it was suddenly buried under 20 feet of ash and pumice when Mount Vesuvius erupted in 79 AD. Despite being covered by volcanic material, Pompeii remained almost perfectly preserved, providing valuable insights into daily life in the Roman Empire.Archaeologists have uncovered a wealth of artifacts, including houses, shops, and even the remains of citizens who were unable to escape the disaster. The city’s layout has also been preserved, showing the streets and buildings much as they were during the 1st century AD. One of the most fascinating aspects of Pompeii is the vivid depiction of everyday life through its artwork andinscriptions.Questions:1.What happened to Pompeii in 79 AD?A) It was destroyed by a fireB) It was buried under volcanic ashC) It was flooded by the seaD) It was attacked by enemies2.Why is Pompeii significant to archaeologists?A) It provides information about the Roman armyB) It reveals the secrets of ancient godsC) It shows the evolution of Roman architectureD) It offers insights into daily life in the Roman Empire3.What are the two ways in which Pompeii has provided insights into everyday life?A) Through the discovery of weapons and armorB) Through the remains of citizens and the city’s layoutC) Through the study of ancient currency and coinsD) Through the analysis of the city’s plumbing systemAnswers:1.B) It was buried under volcanic ash2.D) It offers insights into daily life in the Roman Empire3.B) Through the remains of citizens and the city’s layout五、写作题(16分)Section V: Writing TaskInstructions:Write a short story about a time when you helped someone. Your story should include the following elements:• A clear setting (where and when it happened)•Characters involved•The problem or situation that required help•How you helped and the outcome of your actions•Your feelings before, during, and after helpingYour story should be at least 100 words long. Make sure to use a variety of sentence structures and descriptive language to make your story engaging.Example Answer:One chilly afternoon last winter, I was walking home from school when I noticed Mrs. Thompson, our elderly neighbor, struggling with several heavy grocery bags. She looked tired and her hands were trembling as she tried to unlock her front door. Wit hout a second thought, I rushed over to assist her. “Let me help you with those, Mrs. Thompson,” I offered kindly. She smiled, grateful for the assistance. I carried the groceries into her kitchen and even helped her put them away. After everything was in place, we sat down for a warm cup of tea. Seeing her happy and relieved made my heart swell with joy. It felt wonderful to be able to help someone in need, and it reminded me that small actsof kindness can truly make a big difference.解析:•Setting: 故事设定在一个寒冷的冬日下午,地点是在回家的路上。
人教PEP版英语小学六年级下册期中考试卷6word含答案含答案
加油!有志者事竟成答卷时应注意事项1、拿到试卷,要认真仔细的先填好自己的考生信息。
2、拿到试卷不要提笔就写,先大致的浏览一遍,有多少大题,每个大题里有几个小题,有什么题型,哪些容易,哪些难,做到心里有底;3、审题,每个题目都要多读几遍,不仅要读大题,还要读小题,不放过每一个字,遇到暂时弄不懂题意的题目,手指点读,多读几遍题目,就能理解题意了;容易混乱的地方也应该多读几遍,比如从小到大,从左到右这样的题;4、每个题目做完了以后,把自己的手从试卷上完全移开,好好的看看有没有被自己的手臂挡住而遗漏的题;试卷第1页和第2页上下衔接的地方一定要注意,仔细看看有没有遗漏的小题;5、中途遇到真的解决不了的难题,注意安排好时间,先把后面会做的做完,再来重新读题,结合平时课堂上所学的知识,解答难题;一定要镇定,不能因此慌了手脚,影响下面的答题;6、卷面要清洁,字迹要清工整,非常重要;7、做完的试卷要检查,这样可以发现刚才可能留下的错误或是可以检查是否有漏题,检查的时候,用手指点读题目,不要管自己的答案,重新分析题意,所有计算题重新计算,判断题重新判断,填空题重新填空,之后把检查的结果与先前做的结果进行对比分析。
亲爱的小朋友,你们好! 经过两个月的学习,你们一定有不小的收获吧,用你的自信和智慧,认真答题,相信你一定会闯关成功。
相信你是最棒的!12019-2020学年度第二学期期中质量检测试卷六年级英语试卷考试时间:60分钟满分:100分题号一二三四五六七八九总分得分听力部分:(共40分)一、听录音,选择听到的单词。
(10分)()1. A. that B. than C. tell()2. A. younger B. older C. saw()3. A. 15kg B. 15cm C.50kg()4. A. clean B. climb C. climbed ()5. A. 15kg B. 15cm C.50kg二、听录音,判断对错。
新牛津译林版小学六年级英语上册6A期中测试题(附答案及听力材料)
新牛津译林版小学六年级英语上册6A期中测试题(附答案及听力材料)时间:60分钟学校________ 班级____________ 姓名____________ 得分____________听力20%一、根据所听内容,按所听顺序给下列图标出序号,听两遍。
(5分)()()()()()二、判断所听内容是否与图意相符,相符的打“√”,不相符的打“×”,听两遍。
(5分)()()()()()三、根据你所听到的内容,选择单词、词组、句子,听两遍。
(5分)()1. A.means B.mean C.meet D.man()2. A.keep off B.turn off C.take off D.keep of()3. A.pick up B.give up C.get up D.stand up()4. A.her camera B.she cinema C.her cousin D.his camera()5. A.These are their. B.These are theirs. C. They are there. D. They are theirs.四、根据所听内容选择正确答句,听两遍。
(5分)( )1. A. No,it isn’t. B. Sorry,I don’t know.C. Yes, it’s a public sign.( )2. A.Yes,you should go to bed now. B. No,it isn’t.C. Sorry, you can’t.It’s time to go to bed.( )3. A. Right. B. Yes,please. C. All right.( )4. A. Yes, I can’t. B. No, I can. C. No, I can’t. ( )5. A.It’s a diary. B. I’d like a diary.C. I like a diary.笔试80%一、找出与例词画线部分含有相同读音的单词。
小学六年级上册第六次英语第六单元期中试卷[含答案解释]
小学六年级上册英语第六单元期中试卷[含答案解释]英语试题一、综合题(本题有50小题,每小题2分,共100分.每小题不选、错误,均不给分)1.Which of these is used to eat rice?A. KnifeB. ForkC. SpoonD. Plate2.Jack and his dad are in the garage, fixing a __________ (bicycle). Jack is tightening the __________ (screws) while his dad checks the __________ (brakes). They also check the __________ (tires) to make sure they are full of __________ (air). After they finish, they take the bicycle for a test __________ (ride).3.Which one is used for eating salad?A. ForkB. SpoonC. KnifeD. Plate4.James is at a birthday party. He is playing a game of __________ (1) with his friends. They take turns trying to pin the tail on the __________ (2). Afterward, they have some __________ (3) and __________ (4). James loves the __________ (5) because it has chocolate and strawberries.5.Which of these is a type of transport?A. BicycleB. SpoonC. PlateD. Chair6.They _______ (play / plays) soccer every Sunday.7.Which of these is a type of food?A. BreadB. BookC. ChairD. Spoon8.She _______ (be) in the library when I saw her.st weekend, I __________ (1) to the park with my family. We __________ (2) a picnic and __________ (3) sandwiches. I __________ (4) a ball and __________ (5) with my brother. After that, we __________ (6) a walk and __________ (7) some pictures.10.Which animal has a trunk?A. LionB. ElephantC. GiraffeD. Tiger11.What is the plural form of "box"?A. boxsB. boxiesC. boxesD. box12.What is the opposite of "fast"?A. SlowB. HighC. FastD. Tall13.What do we use to write on paper?A. PenB. ShoeC. PlateD. Spoon14.They _______ (be) at home at the moment.15.Which one is a day of the week?A. WednesdayB. SpoonC. PlateD. Chair16.The children ______ (build) a sandcastle at the beach right now. Yesterday, they ______ (make) a big one, but it ______ (fall) down because the waves ______ (be) too strong. Today, they ______ (try) again, but they ______ (work) together to make it stronger.17.I _______ (be) hungry right now.18.Which of these is used for writing?A. EraserB. PenC. PlateD. Knife19.What is the opposite of "up"?A. DownB. TallC. FastD. Hard20.We _______ (visit) our grandparents last summer. They _______ (live) in a small town by the river. We _______ (have) a great time together. We _______ (go) fishing and _______ (cook) a big meal outdoors.21.Which one is a season of the year?A. TuesdayB. WinterC. CarrotD. Dog22.Which one is a body part?A. FootB. TableC. ForkD. Plate23.Which one is used to play music?A. PianoB. TableC. PenD. Hat24.Which animal is known for its ability to swim?A. DogB. FishC. CatD. Elephant25.This morning, I __________ (forget) to bring my homework to school. When I__________ (realize) it, I __________ (ask) my friend to help me. He __________ (lend) me his homework, so I __________ (not/get) into trouble.26.Which of the following is a shape?A. DogB. SquareC. YellowD. Chair27.What do we use to clean our teeth?A. CombB. ToothbrushC. SpoonD. Pen28.We __________ (1) to the library every week. My sister __________ (2) a book about animals, and I __________ (3) a storybook. After we __________ (4) our books, we __________ (5) back home.29.It’s winter now, so I always wear a ______ when I go outside. I also put on my______ to keep my hands warm. My little brother has a new ______, and he loves to play with it in the snow. Sometimes, we go to the ______ to make snowmen and have snowball fights. Afterward, we drink some hot ______ and eat ______.30.We _______ to the beach next month.31.Which of these is used to play tennis?A. BatB. BallC. RacketD. Net32.We __________ (not/have) any homework today, so we __________ (decide) to go outside and play. We __________ (play) basketball for two hours, and then we__________ (sit) in the park and __________ (talk) about our favorite games.33.Every morning, I wake up early and do some exercises. After that, I go to the kitchen for breakfast. I usually eat __ and drink a cup of milk. Sometimes I also have some fruit like bananas or apples. After breakfast, I get dressed in my school uniform and pack my school bag. Then, I go to school by bus.34.My mother ______ (be) very kind and caring. She ______ (work) as a teacher in a local school. Every morning, she ______ (get) up early, makes breakfast for the family, and then she ______ (teach) her students. I ______ (help) her at home with chores in the evening.35.Which of these is a type of tree?A. OakB. SpoonC. PlateD. Dog36.Which of these animals can fly?A. DogB. CatC. BirdD. Elephant37.Which month comes after July?A. JuneB. MayC. AugustD. September38.Which one is a shape?A. CarB. CircleC. TreeD. House39.I ______ (have) a pet hamster named Snowball. Every day, I ______ (feed) him and ______ (clean) his cage. Sometimes, he ______ (run) on his wheel, and I ______ (watch) him play. I ______ (love) my pet a lot.40.My school __________ (have) a big library. Every week, I __________ (borrow) books from the library. Last week, I __________ (borrow) a book about dinosaurs. It__________ (be) very interesting.41.Choose the sentence with the correct use of an article.A. I saw an dog.B. I saw a apple.C. I saw an apple.D. I saw a elephant.42.Which of these is used for cutting food?A. ForkB. KnifeC. SpoonD. Plate43.Tom and his friends are having a picnic in the park. They sit on a __________ and eat some __________ and __________. After eating, they play a game of __________. Everyone feels __________ because they are having so much fun together.44.We ________ (go) to school by bike.45.My friend Alex __________ (be) very good at drawing. He __________ (draw) pictures every day. Last week, he __________ (draw) a beautiful picture of a sunset. He __________ (show) it to the class, and everyone __________ (think) it was amazing.46.What is the color of grass?A. RedB. GreenC. YellowD. Blue47.I _______ (study) for the English test now.48.My little brother is very curious. He loves asking questions. Yesterday, he asked me, “Why do __ fly south for the winter?” I explained to him that they migrate to find __ food. He was very interested and wanted to learn more about __.49.What do we use to eat rice?A. KnifeB. SpoonC. PlateD. Fork50.My brother _______ a new bicycle last week. He rides it to school every day now. (buy, buys, bought, buying)(答案及解释)。
- 1、下载文档前请自行甄别文档内容的完整性,平台不提供额外的编辑、内容补充、找答案等附加服务。
- 2、"仅部分预览"的文档,不可在线预览部分如存在完整性等问题,可反馈申请退款(可完整预览的文档不适用该条件!)。
- 3、如文档侵犯您的权益,请联系客服反馈,我们会尽快为您处理(人工客服工作时间:9:00-18:30)。
2014 小学英语(闽教版)第六册期中测试卷
(满分100分完卷时间:40分钟)
听力部分(40分)
一、听音,写出你所听到的字母(包括大小写)。
(4分)
1. 2. 3. 4.
二、听音,补全单词。
(8分)
1. se_______d
2. pr_______d
3. t_______ed
4. n_______th
5. th_______d
6.winn _______
7. s _______d
8. sc_______d
三、听音,选择你所听到的单词的序号。
(6分)
()1. A. first B. shirt ()2. A. photo B. phone ()3. A. three B. these ()4. A. catch B. watch ()5. A. ball B. fall ()6. A. door B. floor 四、听音,在正确的图下打“√”。
(8分)
1. ( ) ( )
2. ( ) ( )
3.
4.
( ) ( ) ( ) ( )
五、听音,选择正确的答案。
(6分)
( ) 1. What are you going to take?
A. A hat.
B. Some books.
C. A cat.
( ) 2. Where is the music room?
A. On the 4th floor.
B. On the 5th floor.
C. On the 6th floor.
( ) 3. What are they looking for?
A. Some chicken eggs.
B. Some Easter eggs.
C. Some chocolate eggs.
六、听音,在你所听到的句子前打勾,并在线上写出中文意思。
(8分)
1. ( ) My birthday is February 3th.
2. ( ) I can’t ride my bike. I’m very sad.
( ) My birthday is August 2nd. ( ) A dog is coming. I’m very scared.
______________________________ ______________________________
3. ( ) We’re going for a spring outing.
4. ( ) When is Easter?
( ) We’re going to climb a mountain. ( ) When is Christmas?
______________________________ ______________________________
笔试部分(60分)
一、按照要求写出所缺的字母(包括大小写)。
(10分)
Hh Qq
其中是元音字母的有
二、正确抄写下列英文,注意标点和大小写,并在括号中写出中文意思。
(16分)
1. easter bunny
2. january 1st
( ) ( )
3.the bus is moving
( )
4. I'm going to wear my sports shoes. ( )
三、根据中文完成句子,注意使用单词的正确形式。
(8分)
1.Our classroom is on the ___________floor.(第三)
2.Sally is ___________ ___________.(正在爬山)
3.Linda is ___________.(生气)
4.They are ___________(正在打篮球)basketball.
四、选择题。
(10分)
1. ( ) A:When is Easter? B: It’s in .
A: March B: April C: March and April
2. ( ) A: I have a new doll for my birthday. I’m very .
A: happy B: sad C:tired
3. ( ) Peter is 100 meters.
A: running B: jump C: doing
4. ( ) They are going to a mountain next Sunday.
A: climb B: climbing C: run
5. ( ) The music room is the fourth floor.
A: in B: on C: for
五、读一读,选择正确的答句抄写在横线上。
(10分)
1.Is Chen Ling good at doing long jump?
2.When is Peter’s birthday?
3.What are they going to do?
4.Where is the music room?
5.What is Hu Ping doing now?
六、阅读短文,判断正误。
正确的用T表示,错误的用F表示。
(6分)
Dear Grandma and Grandpa,
I got your Easter card yesterday. Thank you. On Easter morning we had a party in the garden. We looked for the eggs. I got nineteen eggs. Ben got twelve. Kate got two. We had a very good time.
My friends and I went for a spring outing on April 2nd. We climbed a mountain. We took some pictures. Here is one of them. Do you like it?
Happy Easter!
Love,
Sally ( ) 1. Sally got an Easter card from her grandpa and grandma.
( ) 2. On Easter they had a party.
( ) 3. Sally looked for the eggs with Li Hong and Hu Ping.
( ) 4. They got thirty-three eggs together(总共).
( ) 5. On April 1st, Sally went for a spring outing.
( ) 6. They had a good time on April 2nd.
March 4th.
Yes, she is.
It’s on the third floor.
They are going for a Spring Outing. He is doing high jump.
参考答案
听力部分
一.1.Dd 2.Gg 3.Vv 4.Yy
二.1. second 2. proud 3. tired 4. ninth
5.third
6. winner
7. sad
8. scared
三、1. first 2. photo 3. these 4. watch 5. ball 6. floor 四.1.I get a new doll for my birthday. I am very happy.
2.Where is the Teachers’ office? On the third floor.
3.The children are going to look for the eggs.
4.My grandpa is proud of me.
五、1. I'm going to take a hat.
2. It's on the sixth floor.
3. They are looking for some Easter eggs.
六、1.My birthday is February 3rd.
2.A dog is coming. I’m scared.
3.We’re going for a spring outing.
4.When is Easter?
笔试部分
一.Ii Jj Kk Ll Mm Nn Oo Pp
二.(略)
三1.third2.climbing mountains 3.angry 4.playing 四.C A A A B
五.1.Yes, she is. 2.March 4th. 3.They are going for a spring outing.
4.It’s on the third floor.
5.He is doing high jump.
六、TTFTFT。